Anda di halaman 1dari 30

MARCOS VS MANGLAPUS

G.R. No. 88211 September 15 1989


FACTS:
Former President Marcos, after his and his family spent three year exile in Hawaii, USA, sought to return to the
Philippines. The call is about to request of Marcos family to order the respondents to issue travel order to them
and to enjoin the petition of the President's decision to bar their return to the Philippines.

ISSUE:
Whether or not, in the exercise of the powers granted by the Constitution, the President may prohibit the
Marcoses from returning to the Philippines.

RULING:
Yes
According to Section 1, Article VII of the 1987 Constitution: "The executive power shall be vested in the President
of the Philippines." The phrase, however, does not define what is meant by executive power although the same
article tackles on exercises of certain powers by the President such as appointing power during recess of the
Congress (S.16), control of all the executive departments, bureaus, and offices (Section 17), power to grant
reprieves, commutations, and pardons, and remit fines and forfeitures, after conviction by final judgment (Section
19), treaty making power (Section 21), borrowing power (Section 20), budgetary power (Section 22), informing
power (Section 23).
The Constitution may have grant powers to the President, it cannot be said to be limited only to the specific
powers enumerated in the Constitution. Whatever power inherent in the government that is neither legislative nor
judicial has to be executive.

MAXIMO SOLIVEN VS RAMON MAKASIAR


October 29, 2011
167 SCRA 393 – Political Law – Constitutional Law – President’s Immunity From Suit – Must Be Invoked by the
President
Luis Beltran is among the petitioners in this case. He, together with others, was charged with libel by the then
president Corzaon Aquino. Cory herself filed a complaint-affidavit against him and others. Makasiar averred that
Cory cannot file a complaint affidavit because this would defeat her immunity from suit. He grounded his
contention on the principle that a president cannot be sued. However, if a president would sue then the president
would allow herself to be placed under the court’s jurisdiction and conversely she would be consenting to be sued
back. Also, considering the functions of a president, the president may not be able to appear in court to be a
witness for herself thus she may be liable for contempt.
ISSUE: Whether or not such immunity can be invoked by Beltran, a person other than the president.
HELD: No. The rationale for the grant to the President of the privilege of immunity from suit is to assure the
exercise of Presidential duties and functions free from any hindrance or distraction, considering that being the
Chief Executive of the Government is a job that, aside from requiring all of the office-holder’s time, also demands
undivided attention.
But this privilege of immunity from suit, pertains to the President by virtue of the office and may be invoked only
by the holder of the office; not by any other person in the President’s behalf. Thus, an accused like Beltran et al,
in a criminal case in which the President is the complainant cannot raise the presidential privilege as a defense to
prevent the case from proceeding against such accused.
Moreover, there is nothing in our laws that would prevent the President from waiving the privilege. Thus, if so
minded the President may shed the protection afforded by the privilege and submit to the court’s jurisdiction. The
choice of whether to exercise the privilege or to waive it is solely the President’s prerogative. It is a decision that
cannot be assumed and imposed by any other person.

REVIEW CENTER ASSOCIATION OF PHILIPPINES v. EXECUTIVE SECRETARY EDUARDO ERMITA, GR


No. 180046, 2009-04-02
Facts:
The Professional Regulation Commission (PRC) conducted the Nursing Board Examinations nationwide.
handwritten copies of two sets of examinations were circulated during the examination period... among the
examinees reviewing at the R.A. Gapuz Review Center and Inress Review Center
The PRC later admitted the leakage and traced it to two Board of Nursing members.
President Gloria Macapagal-Arroyo (President Arroyo) replaced all the members of the PRC's Board of Nursing.
President Arroyo also ordered the examinees to re-take the Nursing Board Examinations.
President Arroyo issued EO 566 which authorized the CHED to supervise the establishment and operation of all
review centers and similar entities in the Philippines.
the CHED, through its then Chairman Carlito S. Puno (Chairman Puno), approved CHED Memorandum Order
No. 49, series of 2006 (IRR).[... the Review Center Association of the Philippines (petitioner)... that giving permits
to... operate a review center to Higher Education Institutions (HEIs) or consortia of HEIs and professional
organizations will effectively abolish independent review centers.
the OSG alleges that the CHED has the power under paragraphs (e) and (n) of Section 8 to:
The OSG justifies its stand by claiming that the term "programs x x x of higher learning" is broad enough to
include programs offered by review centers.
Its coverage shall be both public and private institutions of higher education as well... as degree-granting
programs in all post-secondary educational institutions, public and private
The OSG argues that President Arroyo was merely exercising her executive power to ensure that the laws are
faithfully executed
292 (EO 292),[29]... particularly Section 20, Title I of Book III, thus:
The exercise of the President's residual powers under this provision requires legislation,[31] as the provision
clearly states... that the exercise of the President's other powers and functions has to be "provided for under the
law."
It is argued that the President of the Philippines has adequate powers under the law to regulate review centers
and this could have been done under an existing validly delegated authority, and that the appropriate law is
Republic Act No. 8981
However, this power should properly be interpreted to... refer to the conduct of the examinations
The PRC has the power to bar PRB members from conducting review classes in review centers... ection 7(y) of
RA 8981 giving the PRC the power to perform "such other functions and duties as may be necessary to carry out
the provisions" of RA 8981 does not extend to the regulation of review centers
Issues:
Whether EO 566 is an unconstitutional exercise by the Executive of legislative power as it expands the CHED's
jurisdiction
Whether the RIRR is an invalid exercise of the Executive's rule-making power
Ruling:
EO 566 Expands the Coverage of RA 7722
We do not agree.
Neither RA 7722 nor CHED Order No. 3, series of 1994 (Implementing Rules of RA 7722)[24] defines an
institution of higher learning or a program of higher learning.
the term "higher education" should be taken in its ordinary sense and should be read and interpreted together
with the phrase "degree-granting programs in all post-secondary educational institutions,... public and private."
The scopes of EO 566 and the RIRR clearly expand the CHED's coverage under RA 7722.
The CHED's coverage under RA 7722 is limited to public and private institutions of higher education and degree-
granting programs in all public and private post-secondary educational... institution
A review center is not an institution of higher learning as contemplated by RA 7722.
The President has no inherent or delegated legislative power to amend the functions of the CHED under RA 7722
Administrative power is concerned with the work of applying policies and enforcing orders as determined by
proper governmental organs. It enables the President to fix a uniform standard of administrative efficiency and
check the official conduct of his agents. To this end, he... can issue administrative orders, rules and regulations.
Republic Act No. 8981 is Not the Appropriate Law
These powers of the PRC have nothing to do at all with the regulation of review centers.
However, to... interpret this power to extend to the power to regulate review centers is clearly an unwarranted
interpretation of RA 8981.
There is absolutely nothing in RA 8981 that mentions regulation by the
PRC of review centers.

PROVINCE OF NORTH COTABATO VS GOVERNMENT OF THE REPUBLIC OF THE PHILIPPINES


FACTS:
On August 5, 2008, the Government of the Republic of the Philippines and the Moro Islamic Liberation Front
(MILF) were scheduled to sign a Memorandum of Agreement of the Ancestral Domain Aspect of the GRP - MILF
Tripoli Agreement on Peace of 2001 in Kuala Lumpur, Malaysia.
Invoking the right to information on matters of public concern, the petitioners seek to compel respondents to
disclose and furnish them the complete and official copies of the MA-AD and to prohibit the slated signing of the
MOA-AD and the holding of public consultation thereon. They also pray that the MOA-AD be declared
unconstitutional. The Court issued a TRO enjoining the GRP from signing the same.
ISSUES:
1. Whether or not the constitutionality and the legality of the MOA is ripe for adjudication;
2. Whether or not there is a violation of the people's right to information on matters of public concern (Art 3 Sec. 7)
under a state policy of full disclosure of all its transactions involving public interest (Art 2, Sec 28) including public
consultation under RA 7160 (Local Government Code of 1991)
3. Whether or not the signing of the MOA, the Government of the Republic of the Philippines would be binding
itself
a) to create and recognize the Bangsamoro Juridical Entity (BJE) as a separate state, or a juridical, territorial or
political subdivision not recognized by law;
b) to revise or amend the Constitution and existing laws to conform to the MOA;
c) to concede to or recognize the claim of the Moro Islamic Liberation Front for ancestral domain in violation of
Republic Act No. 8371 (THE INDIGENOUS PEOPLES RIGHTS ACT OF 1997),
particularly Section 3(g) & Chapter VII (DELINEATION,
RECOGNITION OF ANCESTRAL DOMAINS)
RULINGS:
1. Yes, the petitions are ripe for adjudication. The failure of the respondents to consult the local government units
or communities affected constitutes a departure by respondents from their mandate under EO No. 3. Moreover,
the respondents exceeded their authority by the mere act of guaranteeing amendments to the Constitution. Any
alleged violation of the Constitution by any branch of government is a proper matter for judicial review.
As the petitions involve constitutional issues which are of paramount public interest or of transcendental
importance, the Court grants the petitioners, petitioners-in-intervention and intervening respondents the requisite
locus standi in keeping with the liberal stance adopted in David v. Macapagal- Arroyo.
In Pimentel, Jr. v. Aguirre, this Court held:
x x x [B]y the mere enactment of the questioned law or the approval of the challenged action, the dispute is said
to have ripened into a judicial controversy even without any other overt act . Indeed, even a singular violation of
the Constitution and/or the law is enough to awaken judicial duty.x x x x
By the same token, when an act of the President, who in our constitutional scheme is a coequal of Congress, is
seriously alleged to have infringed the Constitution and the laws x x x settling the dispute becomes the duty and
the responsibility of the courts.
That the law or act in question is not yet effective does not negate ripeness.
2. Yes. The Court finds that there is a grave violation of the Constitution involved in the matters of public concern
(Sec 7 Art III) under a state policy of full disclosure of all its transactions involving public interest (Art 2, Sec 28)
including public consultation under RA 7160 (Local Government Code of 1991).
(Sec 7 ArtIII) The right to information guarantees the right of the people to demand information, while Sec 28
recognizes the duty of officialdom to give information even if nobody demands. The complete and effective
exercise of the right to information necessitates that its complementary provision on public disclosure derive the
same self-executory nature, subject only to reasonable safeguards or limitations as may be provided by law.
The contents of the MOA-AD is a matter of paramount public concern involving public interest in the highest
order. In declaring that the right to information contemplates steps and negotiations leading to the consummation
of the contract, jurisprudence finds no distinction as to the executory nature or commercial character of the
agreement.
E.O. No. 3 itself is replete with mechanics for continuing consultations on both national and local levels and for a
principal forum for consensus-building. In fact, it is the duty of the Presidential Adviser on the Peace Process to
conduct regular dialogues to seek relevant information, comments, advice, and recommendations from peace
partners and concerned sectors of society.

3.a) to create and recognize the Bangsamoro Juridical Entity (BJE) as a separate state, or a juridical, territorial or
political subdivision not recognized by law;
Yes. The provisions of the MOA indicate, among other things, that the Parties aimed to vest in the BJE the status
of an associated state or, at any rate, a status closely approximating it.
The concept of association is not recognized under the present Constitution.
No province, city, or municipality, not even the ARMM, is recognized under our laws as having an “associative”
relationship with the national government. Indeed, the concept implies powers that go beyond anything ever
granted by the Constitution to any local or regional government. It also implies the recognition of the associated
entity as a state. The Constitution, however, does not contemplate any state in this jurisdiction other than the
Philippine State, much less does it provide for a transitory status that aims to prepare any part of Philippine
territory for independence.
The BJE is a far more powerful entity than the autonomous region recognized in the Constitution. It is not merely
an expanded version of the ARMM, the status of its relationship with the national government being fundamentally
different from that of the ARMM. Indeed, BJE is a state in all but name as it meets the criteria of a state laid down
in the Montevideo Convention, namely, a permanent population, a defined territory, a government, and a capacity
to enter into relations with other states.
Even assuming arguendo that the MOA-AD would not necessarily sever any portion of Philippine territory, the
spirit animating it – which has betrayed itself by its use of the concept of association – runs counter to the national
sovereignty and territorial integrity of the Republic.
The defining concept underlying the relationship between the national government and the BJE being itself
contrary to the present Constitution, it is not surprising that many of the specific provisions of the MOA-AD on the
formation and powers of the BJE are in conflict with the Constitution and the laws. The BJE is more of a state
than an autonomous region. But even assuming that it is covered by the term “autonomous region” in the
constitutional provision just quoted, the MOA-AD would still be in conflict with it.

b) to revise or amend the Constitution and existing laws to conform to the MOA:
The MOA-AD provides that “any provisions of the MOA-AD requiring amendments to the existing legal framework
shall come into force upon the signing of a Comprehensive Compact and upon effecting the necessary changes
to the legal framework,” implying an amendment of the Constitution to accommodate the MOA-AD. This
stipulation, in effect, guaranteed to the MILF the amendment of the Constitution .
It will be observed that the President has authority, as stated in her oath of office, only to preserve and defend the
Constitution. Such presidential power does not, however, extend to allowing her to change the Constitution, but
simply to recommend proposed amendments or revision. As long as she limits herself to recommending these
changes and submits to the proper procedure for constitutional amendments and revision, her mere
recommendation need not be construed as an unconstitutional act.
The “suspensive clause” in the MOA-AD viewed in light of the above-discussed standards.
Given the limited nature of the President’s authority to propose constitutional amendments, she cannot guarantee
to any third party that the required amendments will eventually be put in place, nor even be submitted to a
plebiscite. The most she could do is submit these proposals as recommendations either to Congress or the
people, in whom constituent powers are vested.
c) to concede to or recognize the claim of the Moro Islamic Liberation Front for ancestral domain in violation of
Republic Act No. 8371 (THE INDIGENOUS PEOPLES RIGHTS ACT OF 1997),
particularly Section 3(g) & Chapter VII (DELINEATION,
RECOGNITION OF ANCESTRAL DOMAINS)
This strand begins with the statement that it is “the birthright of all Moros and all Indigenous peoples of Mindanao
to identify themselves and be accepted as ‘Bangsamoros.’” It defines “Bangsamoro people” as the natives or
original inhabitants of Mindanao and its adjacent islands including Palawan and the Sulu archipelago at the time
of conquest or colonization, and their descendants whether mixed or of full blood, including their spouses.
Thus, the concept of “Bangsamoro,” as defined in this strand of the MOA-AD, includes not only “Moros” as
traditionally understood even by Muslims, but all indigenous peoples of Mindanao and its adjacent islands. The
MOA-AD adds that the freedom of choice of indigenous peoples shall be respected. What this freedom of choice
consists in has not been specifically defined. The MOA-AD proceeds to refer to the “Bangsamoro homeland,” the
ownership of which is vested exclusively in the Bangsamoro people by virtue of their prior rights of occupation.
Both parties to the MOA-AD acknowledge that ancestral domain does not form part of the public domain.
Republic Act No. 8371 or the Indigenous Peoples Rights Act of 1997 provides for clear-cut procedure for the
recognition and delineation of ancestral domain, which entails, among other things, the observance of the free
and prior informed consent of the Indigenous Cultural Communities/Indigenous Peoples. Notably, the statute does
not grant the Executive Department or any government agency the power to delineate and recognize an ancestral
domain claim by mere agreement or compromise.
Two, Republic Act No. 7160 or the Local Government Code of 1991 requires all national offices to conduct
consultations beforeany project or program critical to the environment and human ecology including those that
may call for the eviction of a particular group of people residing in such locality, is implemented therein. The MOA-
AD is one peculiar program that unequivocally and unilaterally vests ownership of a vast territory to the
Bangsamoro people, which could pervasively and drastically result to the diaspora or displacement of a great
number of inhabitants from their total environment.
CONCLUSION:
In sum, the Presidential Adviser on the Peace Process committed grave abuse of discretion when he failed to
carry out the pertinent consultation process, as mandated by E.O. No. 3, Republic Act No. 7160, and Republic Act
No. 8371. The furtive process by which the MOA-AD was designed and crafted runs contrary to and in excess of
the legal authority, and amounts to a whimsical, capricious, oppressive, arbitrary and despotic exercise thereof. It
illustrates a gross evasion of positive duty and a virtual refusal to perform the duty enjoined.

The MOA-AD cannot be reconciled with the present Constitution and laws. Not only its specific provisions but the
very concept underlying them, namely, the associative relationship envisioned between the GRP and the BJE, are
unconstitutional, for the concept presupposes that the associated entity is a state and implies that the same is on
its way to independence.

BIRAOGO V. PHILIPPINE TRUTH COMMISSION


FACTS:
E.O No. 1 establishing the Philippine Truth Commission (PTC) of 2010 was signed by President Aquino. The said
PTC is a mere branch formed under the Office of the President tasked to investigate reports of graft and
corruption committed by third-level public officers and employees, their co-principals, accomplices and
accessories during the previous administration and submit their findings and recommendations to the President,
Congress and the Ombudsman. However, PTC is not a quasi-judicial body, it cannot adjudicate, arbitrate,
resolve, settle or render awards in disputes between parties. Its job is to investigate, collect and asses evidences
gathered and make recommendations. It has subpoena powers but it has no power to cite people in contempt or
even arrest. It cannot determine for such facts if probable cause exist as to warrant the filing of an information in
our courts of law.
Petitioners contends the Constitutionality of the E.O. on the grounds that.
It violates separation of powers as it arrogates the power of Congress to create a public office and appropriate
funds for its operation;
The provisions of Book III, Chapter 10, Section 31 of the Administrative Code of 1987 cannot legitimize E.O. No. 1
because the delegated authority of the President to structurally reorganize the Office of the President to achieve
economy, simplicity, and efficiency does not include the power to create an entirely new office was inexistent like
the Truth Commission;
The E.O illegally amended the Constitution when it made the Truth Commission and vesting it the power
duplicating and even exceeding those of the Office of the Ombudsman and the DOJ.
It violates the equal protection clause
ISSUE:
WHETHER OR NOT the said E.O is unconstitutional.
RULING:
Yes, E.O No. 1 should be struck down as it is violative of the equal protection clause. The Chief Executive’s
power to create the Ad hoc Investigating Committee cannot be doubted. Having been constitutionally granted full
control of the Executive Department, to which respondents belong, the President has the obligation to ensure that
all executive officials and employees faithfully comply with the law. With AO 298 as mandate, the legality of the
investigation is sustained. Such validity is not affected by the fact that the investigating team and the PCAGC had
the same composition, or that the former used the offices and facilities of the latter in conducting the inquiry.

FUNA VS AGRA
G.R. No. 191644 February 19, 2013
Facts: The petitioner alleges that on March 1, 2010, President Gloria M. Macapagal Arroyo appointed Agra as the
Acting Secretary of Justice following the resignation of Secretary Agnes VST Devanadera in order to vie for a
congressional seat in Quezon Province; that on March 5, 2010, President Arroyo designated Agra as the Acting
Solicitor General in a concurrent capacity; that on April 7, 2010, the petitioner, in his capacity as a taxpayer, a
concerned citizen and a lawyer, commenced this suit to challenge the constitutionality of Agra’s concurrent
appointments or designations, claiming it to be prohibited under Section 13, Article VII of the 1987 Constitution;
that during the pendency of the suit, President Benigno S. Aquino III appointed Atty. Jose Anselmo I. Cadiz as the
Solicitor General; and that Cadiz assumed as the Solicitor General and commenced his duties as such on August
5, 2010. Agra renders a different version of the antecedents. He represents that on January 12, 2010, he was
then the Government Corporate Counsel when President Arroyo designated him as the Acting Solicitor General in
place of Solicitor General Devanadera who had been appointed as the Secretary of Justice; that on March 5,
2010, President Arroyo designated him also as the Acting Secretary of Justice vice Secretary Devanadera who
had meanwhile tendered her resignation in order to run for Congress representing a district in Quezon Province in
the May 2010 elections; that he then relinquished his position as the Government Corporate Counsel; and that
pending the appointment of his successor, Agra continued to perform his duties as the Acting Solicitor General.
Notwithstanding the conflict in the versions of the parties, the fact that Agra has admitted to holding the two
offices concurrently in acting capacities is settled, which is sufficient for purposes of resolving the constitutional
question that petitioner raises herein.
Issue: Whether or not Agra’s holding of concurrent position is unconstitutional.
Held: Yes. At the center of the controversy is the correct application of Section 13, Article VII of the 1987
Constitution, viz:
Section 13. The President, Vice-President, the Members of the Cabinet, and their deputies or assistants
shall not, unless otherwise provided in this Constitution, hold any other office or employment during their
tenure. They shall not, during said tenure, directly or indirectly practice any other profession, participate in
any business, or be financially interested in any contract with, or in any franchise, or special privilege
granted by the Government or any subdivision, agency, or instrumentality thereof, including government-
owned or controlled corporations or their subsidiaries. They shall strictly avoid conflict of interest in the
conduct of their office.
A relevant and complementing provision is Section 7, paragraph (2), Article IX-B of the 1987 Constitution, to wit:
Section 7. x x x Unless otherwise allowed by law or the primary functions of his position, no appointive official
shall hold any other office or employment in the Government or any subdivision, agency or instrumentality thereof,
including government-owned or controlled corporations or their subsidiaries.
Being designated as the Acting Secretary of Justice concurrently with his position of Acting Solicitor General,
therefore, Agra was undoubtedly covered by Section 13, Article VII, supra, whose text and spirit were too clear to
be differently read. Hence, Agra could not validly hold any other office or employment during his tenure as the
Acting Solicitor General, because the Constitution has not otherwise so provided.
It was of no moment that Agra’s designation was in an acting or temporary capacity. The text of Section 13,
supra, plainly indicates that the intent of the Framers of the Constitution was to impose a stricter prohibition on the
President and the Members of his Cabinet in so far as holding other offices or employments in the Government or
in government-owned or government controlled-corporations was concerned. In this regard, to hold an office
means to possess or to occupy the office, or to be in possession and administration of the office, which implies
nothing less than the actual discharge of the functions and duties of the office. Indeed, in the language of Section
13 itself, supra, the Constitution makes no reference to the nature of the appointment or designation. The
prohibition against dual or multiple offices being held by one official must be construed as to apply to all
appointments or designations, whether permanent or temporary, for it is without question that the avowed
objective of Section 13, supra, is to prevent the concentration of powers in the Executive Department officials,
specifically the President, the Vice-President, the Members of the Cabinet and their deputies and assistants. To
construe differently is to “open the veritable floodgates of circumvention of an important constitutional
disqualification of officials in the Executive Department and of limitations on the Presidents power of appointment
in the guise of temporary designations of Cabinet Members, undersecretaries and assistant secretaries as
officers-in-charge of government agencies, instrumentalities, or government-owned or controlled corporations.
It is not amiss to observe, lastly, that assuming that Agra, as the Acting Solicitor General, was not covered by the
stricter prohibition under Section 13, supra, due to such position being merely vested with a cabinet rank under
Section 3, Republic Act No. 9417, he nonetheless remained covered by the general prohibition under Section 7,
supra. Hence, his concurrent designations were still subject to the conditions under the latter constitutional
provision. In this regard, the Court aptly pointed out in Public Interest Center, Inc. v. Elma:

The general rule contained in Article IX-B of the 1987 Constitution permits an appointive official to hold more than
one office only if “allowed by law or by the primary functions of his position.” In the case of Quimson v. Ozaeta,
this Court ruled that, “[t]here is no legal objection to a government official occupying two government offices and
performing the functions of both as long as there is no incompatibility.” The crucial test in determining whether
incompatibility exists between two offices was laid out in People v. Green – whether one office is subordinate to
the other, in the sense that one office has the right to interfere with the other.

CIVIL LIBERTIES UNION VS EXECUTIVE SECRETARY


194 SCRA 317 – Political Law – Ex Officio Officials – Members of the Cabinet – Singularity of Office – EO 284
In July 1987, then President Corazon Aquino issued Executive Order No. 284 which allowed members of the
Cabinet, their undersecretaries and assistant secretaries to hold other government offices or positions in addition
to their primary positions subject to limitations set therein. The Civil Liberties Union (CLU) assailed this EO
averring that such law is unconstitutional. The constitutionality of EO 284 is being challenged by CLU on the
principal submission that it adds exceptions to Sec 13, Article 7 of the Constitution which provides:
“Sec. 13. The President, Vice-President, the Members of the Cabinet, and their deputies or assistants shall not,
unless otherwise provided in this Constitution, hold any other office or employment during their tenure. They shall
not, during said tenure, directly or indirectly practice any other profession, participate in any business, or be
financially interested in any contract with, or in any franchise, or special privilege granted by the Government or
any subdivision, agency, or instrumentality thereof, including government-owned or controlled corporations or
their subsidiaries. They shall strictly avoid conflict of interest in the conduct of their office.”
CLU avers that by virtue of the phrase “unless otherwise provided in this Constitution“, the only exceptions
against holding any other office or employment in Government are those provided in the Constitution, namely: (i)
The Vice-President may be appointed as a Member of the Cabinet under Sec 3, par. (2), Article 7; and (ii) the
Secretary of Justice is an ex-officio member of the Judicial and Bar Council by virtue of Sec 8 (1), Article 8.
ISSUE: Whether or not EO 284 is constitutional.
HELD: No, it is unconstitutional. It is clear that the 1987 Constitution seeks to prohibit the President, Vice-
President, members of the Cabinet, their deputies or assistants from holding during their tenure multiple offices or
employment in the government, except in those cases specified in the Constitution itself and as above clarified
with respect to posts held without additional compensation in an ex-officio capacity as provided by law and as
required by the primary functions of their office, the citation of Cabinet members (then called Ministers) as
examples during the debate and deliberation on the general rule laid down for all appointive officials should be
considered as mere personal opinions which cannot override the constitution’s manifest intent and the people’s
understanding thereof.
In the light of the construction given to Sec 13, Art 7 in relation to Sec 7, par. (2), Art IX-B of the 1987
Constitution, EO 284 is unconstitutional. Ostensibly restricting the number of positions that Cabinet members,
undersecretaries or assistant secretaries may hold in addition to their primary position to not more than 2
positions in the government and government corporations, EO 284 actually allows them to hold multiple offices or
employment in direct contravention of the express mandate of Sec 13, Art 7 of the 1987 Constitution prohibiting
them from doing so, unless otherwise provided in the 1987 Constitution itself.

DOMINADOR AYTONA VS ANDRES CASTILLO


4 SCRA 1 – Political Law – Appointing Power – Midnight Appointments
Dominador Aytona was one of those appointed by outgoing president Carlos Garcia during the last day of his
term. Aytona was appointed as the ad interim governor of the Central Bank. When the next president, Diosdado
Macapagal took his office, he issued Order No. 2 which recalled Aytona’s position and at the same time he
appointed Andres Castillo as the new governor of the Central Bank. Aytona then filed a quo warranto proceeding
claiming that he is qualified to remain as the Central Bank governor and that he was validly appointed by the
former president. Macapagal averred that the ex-president’s appointments were scandalous, irregular, hurriedly
done, contrary to law and the spirit of which, and it was an attempt to subvert the incoming presidency or
administration.
ISSUE: Whether or not Aytona should remain in his post.
HELD: No. Had the appointment of Aytona been done in good faith then he would have the right to continue
office. Here, even though Aytona is qualified to remain in his post as he is competent enough, his appointment
can nevertheless be revoked by the president. Garcia’s appointments are hurried maneuvers to subvert the
upcoming administration and is set to obstruct the policies of the next president. As a general rule, once a person
is qualified his appointment should not be revoked but in here it may be since his appointment was grounded on
bad faith, immorality and impropriety. In public service, it is not only legality that is considered but also justice,
fairness and righteousness.

IN RE APPOINTMENTS OF HON. MATEO VALENZUELA AND HON. PLACIDO VALLARTA A.M. No. 98-5-01-
SC, November 9, 1998
Facts: Referred to the Court en banc are the appointments signed by the President dated March 30, 1998 of Hon.
Mateo Valenzuela and Hon. Placido Vallarta as judges of the RTC of Bago City and Cabanatuan City,
respectively. These appointments appear prima facie, at least, to be expressly prohibited by Sec. 15, Art. VII of
the Constitution. The said constitutional provision prohibits the President from making any appointments two
months immediately before the next presidential elections and up to the end of his term, except temporary
appointments to executive positions when continued vacancies therein will prejudice public service or endanger
public safety.
Issue: Whether or not, during the period of the ban on appointments imposed by Sec. 15, Art. VII of the
Constitution, the President is nonetheless required to fill vacancies in the judiciary, in view of Secs. 4 (1) and 9 of
Art. VIII
Held: During the period stated in Sec. 15, Art. VII of the Constitution “two months immediately before the next
presidential elections and up to the end of his term” the President is neither required to make appointments to the
courts nor allowed to do so; and that Secs. 4(1) and 9 of Art. VIII simply mean that the President is required to fill
vacancies in the courts within the time frames provided therein unless prohibited by Sec. 15 of Art. VII. This
prohibition on appointments comes into effect once every 6 years.
The appointments of Valenzuela and Vallarta were unquestionably made during the period of the ban. They come
within the operation of the prohibition relating to appointments. While the filling of vacancies in the judiciary is
undoubtedly in the public interest, there is no showing in this case of any compelling reason to justify the making
of the appointments during the period of the ban.

ARTURO M. DE CASTRO vs. JUDICIAL AND BAR COUNCIL (JBC) and PRESIDENT GLORIA MACAPAGAL
– ARROYO
G.R. No. 191002, March 17, 2010
FACTS: The compulsory retirement of Chief Justice Reynato S. Puno by May 17, 2010 occurs just days after the
coming presidential elections on May 10, 2010.
These cases trace their genesis to the controversy that has arisen from the forthcoming compulsory retirement of
Chief Justice Puno on May 17, 2010, or seven days after the presidential election. Under Section 4(1), in relation
to Section 9, Article VIII, that “vacancy shall be filled within ninety days from the occurrence thereof” from a “list of
at least three nominees prepared by the Judicial and Bar Council for every vacancy.” Also considering that
Section 15, Article VII (Executive Department) of the Constitution prohibits the President or Acting President from
making appointments within two months immediately before the next presidential elections and up to the end of
his term, except temporary appointments to executive positions when continued vacancies therein will prejudice
public service or endanger public safety.

The JBC, in its en banc meeting of January 18, 2010, unanimously agreed to start the process of filling up the
position of Chief Justice.
Conformably with its existing practice, the JBC “automatically considered” for the position of Chief Justice the five
most senior of the Associate Justices of the Court, namely: Associate Justice Antonio T. Carpio; Associate Justice
Renato C. Corona; Associate Justice Conchita Carpio Morales; Associate Justice Presbitero J. Velasco, Jr.; and
Associate Justice Antonio Eduardo B. Nachura. However, the last two declined their nomination through letters
dated January 18, 2010 and January 25, 2010, respectively.
The OSG contends that the incumbent President may appoint the next Chief Justice, because the prohibition
under Section 15, Article VII of the Constitution does not apply to appointments in the Supreme Court. It argues
that any vacancy in the Supreme Court must be filled within 90 days from its occurrence, pursuant to Section 4(1),
Article VIII of the Constitution; that had the framers intended the prohibition to apply to Supreme Court
appointments, they could have easily expressly stated so in the Constitution, which explains why the prohibition
found in Article VII (Executive Department) was not written in Article VIII (Judicial Department); and that the
framers also incorporated in Article VIII ample restrictions or limitations on the President’s power to appoint
members of the Supreme Court to ensure its independence from “political vicissitudes” and its “insulation from
political pressures,” such as stringent qualifications for the positions, the establishment of the JBC, the specified
period within which the President shall appoint a Supreme Court Justice.
A part of the question to be reviewed by the Court is whether the JBC properly initiated the process, there being
an insistence from some of the oppositors-intervenors that the JBC could only do so once the vacancy has
occurred (that is, after May 17, 2010). Another part is, of course, whether the JBC may resume its process until
the short list is prepared, in view of the provision of Section 4(1), Article VIII, which unqualifiedly requires the
President to appoint one from the short list to fill the vacancy in the Supreme Court (be it the Chief Justice or an
Associate Justice) within 90 days from the occurrence of the vacancy.
ISSUE: Whether the incumbent President can appoint the successor of Chief Justice Puno upon his retirement.
HELD:
Prohibition under Section 15, Article VII does not apply to appointments to fill a vacancy in the Supreme Court or
to other appointments to the Judiciary.
Two constitutional provisions are seemingly in conflict.
The first, Section 15, Article VII (Executive Department), provides: Section 15. Two months immediately before
the next presidential elections and up to the end of his term, a President or Acting President shall not make
appointments, except temporary appointments to executive positions when continued vacancies therein will
prejudice public service or endanger public safety.
The other, Section 4 (1), Article VIII (Judicial Department), states: Section 4. (1). The Supreme Court shall be
composed of a Chief Justice and fourteen Associate Justices. It may sit en banc or in its discretion, in division of
three, five, or seven Members. Any vacancy shall be filled within ninety days from the occurrence thereof.
Had the framers intended to extend the prohibition contained in Section 15, Article VII to the appointment of
Members of the Supreme Court, they could have explicitly done so. They could not have ignored the meticulous
ordering of the provisions. They would have easily and surely written the prohibition made explicit in Section 15,
Article VII as being equally applicable to the appointment of Members of the Supreme Court in Article VIII itself,
most likely in Section 4 (1), Article VIII. That such specification was not done only reveals that the prohibition
against the President or Acting President making appointments within two months before the next presidential
elections and up to the end of the President’s or Acting President’s term does not refer to the Members of the
Supreme Court.
Had the framers intended to extend the prohibition contained in Section 15, Article VII to the appointment of
Members of the Supreme Court, they could have explicitly done so. They could not have ignored the meticulous
ordering of the provisions. They would have easily and surely written the prohibition made explicit in Section 15,
Article VII as being equally applicable to the appointment of Members of the Supreme Court in Article VIII itself,
most likely in Section 4 (1), Article VIII. That such specification was not done only reveals that the prohibition
against the President or Acting President making appointments within two months before the next presidential
elections and up to the end of the President’s or Acting President’s term does not refer to the Members of the
Supreme Court.
Section 14, Section 15, and Section 16 are obviously of the same character, in that they affect the power of the
President to appoint. The fact that Section 14 and Section 16 refer only to appointments within the Executive
Department renders conclusive that Section 15 also applies only to the Executive Department. This conclusion is
consistent with the rule that every part of the statute must be interpreted with reference to the context, i.e. that
every part must be considered together with the other parts, and kept subservient to the general intent of the
whole enactment. It is absurd to assume that the framers deliberately situated Section 15 between Section 14 and
Section 16, if they intended Section 15 to cover all kinds of presidential appointments. If that was their intention in
respect of appointments to the Judiciary, the framers, if only to be clear, would have easily and surely inserted a
similar prohibition in Article VIII, most likely within Section 4 (1) thereof.

SARMIENTO III VS MISON AND CARAGUE


156 SCRA 549 G.R. No. 79974 December 17 1987 [Appointing Power]
FACTS:
Mison was appointed as the Commissioner of the Bureau of Customs and Carague as the Secretary of the
Department of Budget, without the confirmation of the Commission on Appointments. Sarmiento assailed the
appointments as unconstitutional by reason of its not having been confirmed by CoA.
ISSUE:
Whether or not the appointment is valid.
RULING:
Yes. The President acted within her constitutional authority and power in appointing Salvador Mison, without
submitting his nomination to the CoA for confirmation. He is thus entitled to exercise the full authority and
functions of the office and to receive all the salaries and emoluments pertaining thereto.
Under Sec 16 Art. VII of the 1987 Constitution, there are 4 groups of officers whom the President shall appoint:
1st, appointment of executive departments and bureaus heads, ambassadors, other public ministers, consuls,
officers of the armed forces from the rank of colonel or naval captain, and other officers with the consent and
confirmation of the CoA.
2nd, all other Government officers whose appointments are not otherwise provided by law;
3rd those whom the President may be authorized by the law to appoint;
4th, low-ranking officers whose appointments the Congress may by law vest in the President alone.
First group of officers is clearly appointed with the consent of the Commission on Appointments. Appointments of
such officers are initiated by nomination and, if the nomination is confirmed by the Commission on Appointments,
the President appoints.
2nd, 3rd and 4th group of officers are the present bone of contention. By following the accepted rule in
constitutional and statutory construction that an express enumeration of subjects excludes others not
enumerated, it would follow that only those appointments to positions expressly stated in the first group require
the consent (confirmation) of the Commission on Appointments.
It is evident that the position of Commissioner of the Bureau of Customs (a bureau head) is not one of those
within the first group of appointments where the consent of the Commission on Appointments is required. The
1987 Constitution deliberately excluded the position of "heads of bureaus" from appointments that need the
consent (confirmation) of the Commission on Appointments.
MARY CONCEPCION-BAUTISTA VS SENATOR JOVITO SALONGA
October 31, 2011
172 SCRA 150 – Political Law – Appointments – Commission on Human Rights – Security of Tenure
In August 1987, then President Corazon Aquino designated Mary Concepcion-Bautista as the Acting Chairwoman
of Commission on Human Rights. In December 1987, Cory made the designation of Bautista permanent.
Bautista then took her oath of office.
Later however, Bautista received a letter from the Commission on Appointments (COA) requiring her to submit
certain documents for her qualification and for confirmation by the COA. Bautista then wrote a letter to the COA
Chairman, Senate President Jovito Salonga, and she explained that her position as chairwoman of the CHR does
not require confirmation by the COA as laid down in the case of Sarmiento vs Mison.
Meanwhile, pending the issue of Bautista’s appointment with the COA, Cory designated Hesiquio Mallilin as the
acting chairman of the CHR.
In 1989, the COA finally disapproved the appointment of Bautista. COA considered Bautista’s appointment as “ad
interim”.
Bautista went to the Supreme Court and questioned COA’s actions. She impleaded Mallillin. Mallillin on his part
invoked Executive Order No. 163-A which provided that the appointment of the CHR chair is at the pleasure of the
president. Hence, since Cory left the issue with the COA and the latter decided not to confirm Bautista, Mallillin
should be allowed to take his seat as chairman of the CHR.
ISSUE: Whether or not Bautista’s appointment is subject to COA’s confirmation.
HELD: No. The appointment of the Chairman and Members of the CHR is not specifically provided for in the
Constitution itself, unlike the Chairmen and Members of the Civil Service Commission, the Commission on
Elections and the Commission on Audit, whose appointments are expressly vested by the Constitution in the
President with the consent of the COA. The President appoints the Chairman and Members of the CHR pursuant
to the second sentence in Sec 16, Art. 7, that is, without the confirmation of the COA because they are among the
officers of government “whom he (the President) may be authorized by law to appoint.” The law which authorizes
the president to make appointments to the CHR is Executive Order No. 163.
The act of Cory submitting Bautista’s appointment to the COA for confirmation is merely political in nature and it
has no basis in law or in the constitution. Appointment to the CHR should be made without the participation of the
COA. Thus, Cory’s act of submitting the appointment of Bautista to the CHR is done without or in excess of
jurisdiction.
Even assuming arguendo that the President can submit such appointment to the COA for the latter’s approval or
rejection, such submission is not valid because at the time of submission, the office of the chairman (chairwoman)
of the CHR is not vacant – as at that time, Bautista already took her oath and was the incumbent CHR
chairperson.
There is also no basis for the COA to consider Bautista’s appointment as “ad interim”. Since the position of
chairman and members of the CHR are not subject to COA confirmation, all appointments to the CHR are always
permanent and cannot be ad interim.
Anent the argument of Mallillin that EO 163-A provides that the chairman and members of the CHR may be
removed at the pleasure of the president, the same is not valid. Thus, EO 163-A is unconstitutional. Note that the
earlier EO 163 provides that the chairman and the members of the CHR shall have a term of 7 years. The
Chairman and the Commissioners of the CHR cannot be removed at the pleasure of the president for it is
guaranteed that they must have a term of office. They can only be removed upon cause and with the observance
of due process.

TERESITA QUINTOS-DELES VS COMMISSION ON CONSTITUTIONAL COMMISSIONS


October 31, 2011
177 SCRA 259 – Political Law – Appointment of Sectoral Representatives
Teresita Quintos-Deles was appointed by then President Corazon Aquino as a sectoral representative for women
in 1988. Their appointment was done while Congress was in session. They were subsequently scheduled to take
their oath of office but the Commission on Appointments (COA) filed an opposition against Deles et al alleging
that their appointment must have the concurrence of the COA.
Deles then questioned the objection of the COA. She said that her appointment does not need the concurrence of
the COA. This is in pursuant to Section 7, Article XVIII of the Constitution, which does not require her appointment
to be confirmed by the COA to qualify her to take her seat in the lower house.
ISSUE: Whether the Constitution requires the appointment of sectoral representatives to the House of
Representatives should be confirmed by the Commission on Appointments.
HELD: Yes. There are four (4) groups of officers whom the President shall appoint. These four (4) groups, to
which we will hereafter refer from time to time, are:
First, the heads of the executive departments, ambassadors, other public ministers and consuls, officers of the
armed forces from the rank of colonel or naval captain, and other officers whose appointments are vested in him
in this Constitution;
Second, all other officers of the Government whose appointments are not otherwise provided for by law;
Third, those whom the President may be authorized by law to appoint;
Fourth, officers lower in rank whose appointments the Congress may by law vest in the President alone.
Only those appointments expressly mentioned in the first sentence of Sec. 16, Art. VII (or the first group
abovementioned) are to be reviewed by the COA, namely, ‘the heads of the executive departments,
ambassadors, other public ministers and consuls, or officers of the armed forces from the rank of colonel or naval
captain, and other officers whose appointments are vested in him in this Constitution.’ All other appointments by
the President are to be made without the participation of the Commission on Appointments.
Sectoral representatives belong to the phrase “and other officers whose appointments are vested in him in this
Constitution“. The provision of the Constitution which provides power to the president in this regard is Section 7,
Article XVII of the 1987 Constitution:
Until a law is passed, the President may fill by appointment from a list of nominees by the respective sectors the
seats reserved for sectoral representation in paragraph (1), Section 5 of Article VI of this Constitution.

PETER JOHN CALDERON VS BARTOLOME CARALE


October 31, 2011
208 SCRA 254 – Political Law – Appointment – List of Appointees Requiring COA Confirmation Cannot Be
Expanded by Law
In 1989, Republic Act No. 6715 was passed. This law amended PD 442 or the Labor Code. RA 6715 provides
that the Chairman, the Division Presiding Commissioners and other Commissioners [of the NLRC] shall all be
appointed by the President, subject to confirmation by the Commission on Appointments (COA).
Pursuant to the said law, President Corazon Aquino appointed Bartolome Carale et al as the Chairman and the
Commissioners respectively of the NLRC. The appointments were however not submitted to the CoA for its
confirmation. Peter John Calderon questioned the appointment saying that without the confirmation by the CoA,
such an appointment is in violation of RA 6715. Calderon insisted that RA 6715 should be followed as he asserted
that RA 6715 is not an encroachment on the appointing power of the executive contained in Sec. 16, Art. 7, of the
Constitution, as Congress may, by law, require confirmation by the Commission on Appointments of other officers
appointed by the President in addition to those mentioned in the first sentence of Sec. 16 of Article 7 of the
Constitution.
ISSUE: Whether or not Congress may, by law, expand the list of public officers required to be confirmed by the
Commission on Appointment as listed in the Constitution.
HELD: No. Under the provisions of the 1987 Constitution, there are four (4) groups of officers whom the
President shall appoint. These four (4) groups are:
First, the heads of the executive departments, ambassadors, other public ministers and consuls, officers of the
armed forces from the rank of colonel or naval captain, and other officers whose appointments are vested in him
in this Constitution;
Second, all other officers of the Government whose appointments are not otherwise provided for by law;
Third, those whom the President may be authorized by law to appoint;
Fourth, officers lower in rank whose appointments the Congress may by law vest in the President alone.
The Supreme Court agreed with the Solicitor General: confirmation by the CoA is required exclusively for the
heads of executive departments, ambassadors, public ministers, consuls, officers of the armed forces from the
rank of colonel or naval captain, and other officers whose appointments are vested in the President by the
Constitution, such as the members of the various Constitutional Commissions (first group). With respect to the
other officers (second to fourth group) whose appointments are not otherwise provided for by the law and to those
whom the President may be authorized by law to appoint, no confirmation by the Commission on Appointments is
required.
“Had it been the intention to allow Congress to expand the list of officers whose appointments must be confirmed
by the Commission on Appointments, the Constitution would have said so by adding the phrase “and other
officers required by law” at the end of the first sentence, or the phrase, “with the consent of the Commission on
Appointments” at the end of the second sentence. Evidently, our Constitution has significantly omitted to provide
for such additions.
This jurisprudence established the following in interpreting Sec 16, Art 7 of the Constitution
1. Confirmation by the Commission on Appointments is required only for presidential appointees mentioned in the
first sentence of Section 16, Article VII, including, those officers whose appointments are expressly vested by the
Constitution itself in the president (like sectoral representatives to Congress and members of the constitutional
commissions of Audit, Civil Service and Election).
2. Confirmation is not required when the President appoints other government officers whose appointments are
not otherwise provided for by law or those officers whom he may be authorized by law to appoint (like the
Chairman and Members of the Commission on Human Rights). Also, as observed in Mison, when Congress
creates inferior offices but omits to provide for appointment thereto, or provides in an unconstitutional manner for
such appointments, the officers are considered as among those whose appointments are not otherwise provided
for by law.

FLORES V DRILON
FACTS
Petitioners, taxpayers and employees of U.S facilities at Subic, challenge the constitutionality of Sec. 13 (d) of the
Bases Conversion and Development Act of 1992 which directs the President to appoint a professional manager
as administrator of the SBMA…provided that “for the 1st year of its operations, the mayor of Olongapo City
(Richard Gordon) shall be appointed as the chairman and the CEO of the Subic Authority.”
ISSUES
(1) Whether the proviso violates the constitutional proscription against appointment or designation of elective
officials to other government posts.

(2) Whether or not the SBMA posts are merely ex officio to the position of Mayor of Olongapo City and thus an
excepted circumstance.
(3) Whether or not the Constitutional provision allowing an elective official to receive double compensation (Sec.
8, Art. IX-B) would be useless if no elective official may be appointed to another post.
(4) Whether there is legislative encroachment on the appointing authority of the President.
(5) Whether Mayor Gordon may retain any and all per diems, allowances and other emoluments which he may
have received pursuant to his appointment.
HELD
(1) YES, Sec. 7 of Art. IX-B of the Constitution Provides: No elective official shall be eligible for appointment or
designation in any capacity to any public office or position during his tenure. Unless otherwise allowed by law or
by the primary functions of his position, no appointive official shall hold any other office or employment in the
Government or any subdivision, agency or instrumentality thereof, including government-owned or controlled
corporations or their subsidiaries. The subject proviso directs the President to appoint an elective official i.e. the
Mayor of Olongapo City, to other government post (as Chairman and CEO of SBMA). This is precisely what the
Constitution prohibits. It seeks to prevent a situation where a local elective official will work for his appointment in
an executive position in government, and thus neglect his constitutents.
(2) NO, Congress did not contemplate making the SBMA posts as automatically attached to the Office of the
Mayor without need of appointment. The phrase “shall be appointed” unquestionably shows the intent to make the
SBMA posts appointive and not merely adjunct to the post of Mayor of Olongapo City.
(3) NO, Sec. 8 does not affect the constitutionality of the subject proviso. In any case, the Vice-President for
example, an elective official who may be appointed to a cabinet post, may receive the compensation attached to
the cabinet position if specifically authorized by law.
(4) YES, although Section 13(d) itself vests in the President the power to appoint the Chairman of SBMA, he
really has no choice but to appoint the Mayor of Olongapo City. The power of choice is the heart of the power to
appoint. Appointment involves an exercise of discretion of whom to appoint. Hence, when Congress clothes the
President with the power to appoint an officer, it cannot at the same time limit the choice of the President to only
one candidate. Such enactment effectively eliminates the discretion of the appointing power to choose and
constitutes an irregular restriction on the power of appointment. While it may be viewed that the proviso merely
sets the qualifications of the officer during the first year of operations of SBMA, i.e., he must be the Mayor of
Olongapo City, it is manifestly an abuse of congressional authority to prescribe qualifications where only one, and
no other, can qualify. Since the ineligibility of an elective official for appointment remains all throughout his tenure
or during his incumbency, he may however resign first from his elective post to cast off the constitutionally-
attached disqualification before he may be considered fit for appointment. Consequently, as long as he is an
incumbent, an elective official remains ineligible for appointment to another public office.
(5) YES, as incumbent elective official, Gordon is ineligible for appointment to the position of Chairman and CEO
of SBMA; hence, his appointment thereto cannot be sustained. He however remains Mayor of Olongapo City, and
his acts as SBMA official are not necessarily null and void; he may be considered a de facto officer, and in
accordance with jurisprudence, is entitled to such benefits.

LUEGO VS CSC, 143 SCRA 327


(Public Officer, Appointments, CSC)
Facts: Petitioner was appointed Admin Officer II, Office of the City Mayor, Cebu City, by Mayor Solon. The
appointment was described as “permanent” but the CSC approved it as “temporary,” subject to the final action
taken in the protest filed by the private respondent and another employee.
Subsequently, the CSC found the private respondent better qualified than the petitioner for the contested position
and, accordingly directed that the latter be appointed to said position in place of the petitioner whose appointment
is revoked. Hence, the private respondent was so appointed to the position by Mayor Duterte, the new mayor.
The petitioner, invoking his earlier permanent appointment, questions the order and the validity of the
respondent’s appointment.
Issue: WON the CSC is authorized to disapprove a permanent appointment on the ground that another person is
better qualified than the appointee and, on the basis of this finding, order his replacement.
Held: No. The appointment of the petitioner was not temporary but permanent and was therefore protected by
Constitution. The appointing authority indicated that it was permanent, as he had the right to do so, and it was not
for the respondent CSC to reverse him and call it temporary.
Section 9(h), Art V of the Civil Service Decree provides that the Commission shall have inter alia the power to
“…approve all appointments, whether original or promotional, to positions in the civil service… ….and disapprove
those where the appointees do not possess appropriate eligibility or required qualifications.”
The CSC is not empowered to determine the kind or nature of the appointment extended by the appointing officer,
its authority being limited to approving or reviewing the appointment in the light of the requirements of the CSC
Law. When the appointee is qualified and all the other legal requirements are satisfied, the Commission has no
choice but to attest to the appointment in accordance with the CSC Laws.
CSC is without authority to revoke an appointment because of its belief that another person was better qualified,
which is an encroachment on the discretion vested solely in the city mayor.

Section 16. The President shall nominate and, with the consent of the Commission on Appointments, appoint the
heads of the executive departments, ambassadors, other public ministers and consuls, or officers of the armed
forces from the rank of colonel or naval captain, and other officers whose appointments are vested in him in this
Constitution. He shall also appoint all other officers of the Government whose appointments are not otherwise
provided for by law, and those whom he may be authorized by law to appoint. The Congress may, by law, vest
the appointment of other officers lower in rank in the President alone, in the courts, or in the heads of
departments, agencies, commissions, or boards.
The President shall have the power to make appointments during the recess of the Congress, whether voluntary
or compulsory, but such appointments shall be effective only until disapproved by the Commission on
Appointments or until the next adjournment of the Congress.
POBRE VS. MENDIETA [G.R. No. 106677, July 23, 1993]
FACTS:These consolidated petitions under Rules 45 and 65 of the Rules of Court were filed by Her genes Pobre
to set aside the court’s decision and writ of prohibitory injunction, issued by Judge Corona Ibay-Somera, annulling
the appointment extended by President Corazon C. Aquino to the petitioner, HermogenesPobre, as
Commissioner/Chairman of the Professional Regulation Commission mo (PRC) and enjoining him from
discharging the duties and functions of that office.
The controversy began on January 2, 1992, when the term of office of Honorable Julio B. Francia as PRC
Commissioner/Chairman expired. At that time, Mariano A. Mendieta was the senior Associate Commissioner and
Hermogenes P. Pobre was the second Associate Commissioner of the PRC.
The Executive Secretary wanted to know whether the President may appoint as Commissioner/Chairman of the
PRC another Associate Commissioner or any person other than the Senior Associate Commissioner.
In a Memorandum, Acting Secretary of Justice Silvestre H. Bello, III answered the queries as follows: Based on
the foregoing premises, it is our view that Section 2 of P.D. No. 223 does not limit or restrict the appointing power
of the President. It has been said that "those matters which the Constitution specifically confides to the executive,
the legislative cannot directly or indirectly take from his control"
Pobre opposed the issuance of a restraining order because President Aquino had already appointed him PRC
Chairman and he had, in fact, already taken his oath of office on February 17, 1992. Judge Somera denied the
prayer for a restraining order as well as the petition for declaratory relief for being moot and academic.
Consequently, Mendieta filed a petition for quo warranto contesting Pobre's appointment as chairman of the PRC
because he (Mendieta) allegedly succeeded Francia as PRC Chairman by operation of law. Pobre disputed
Mendieta's claim on the ground that only the President of the Philippines, in whom the appointing power is vested
by law and the Constitution, may name the successor of retired PRC Commissioner/Chairman Francia upon the
expiration of the latter's term of office.
The petition raises an issue regarding the proper construction of the provision in Section 2 of P.D. No. 223 that: ".
. . any vacancy in the Commission shall be filled for the unexpired term only with the most Senior of the Associate
Commissioners succeeding the Commissioner at the expiration of his term, resignation or removal," whereby the
legality of Pobre's appointment as PRC Chairman may be determined.
In interpreting this section of P.D. No. 223, consideration should be accorded the provision of the Constitution
vesting the power of appointment in the President of the Philippines.
ISSUE: Whether or not the appointment of Pobre as Commissioner/Chairman of the Professional Regulation
Commission by the President is lawful.
RULING: THE POWER OF APPOINTMENT CANNOT BE RESTRICTED TO THE POINT THAT THE OFFICER
LOSES THE DISCRETION. The Court finds unacceptable the view that every vacancy in the Commission
(except the position of "junior" Associate Commissioner) shall be filled by "succession" or by "operation of law" for
that would deprive the President of his power to appoint a new PRC Commissioner and Associate Commissioners
— "all to be appointed by the President" under P.D. No. 223. The absurd result would be that the only occasion
for the President to exercise his appointing power would be when the position of junior (or second) Associate
Commissioner becomes vacant. We may not presume that when the President issued P.D. No. 223, he
deliberately clipped his prerogative to choose and appoint the head of the PRC and limited himself to the
selection and appointment of only the associate commissioner occupying the lowest rung of the ladder in that
agency. Since such an absurdity may not be presumed, the Court should so construe the law as to avoid it.
"The duty devolves on the court to ascertain the true meaning where the language of a statute is of doubtful
meaning, or where an adherence to the strict letter would lead to injustice, absurdity, or contradictory provisions,
since an ambiguity calling for construction may arise when the consequence of a literal interpretation of the
language is an unjust, absurd, unreasonable, or mischievous result, or one at variance with the policy of the
legislation as a whole; and the real meaning of the statute is to be ascertained and declared, even though it
seems to conflict with the words of the statute." (82 CJS 589-590; Emphasis supplied.)

ARTURO M. DE CASTRO vs. JUDICIAL AND BAR COUNCIL (JBC) and PRESIDENT GLORIA MACAPAGAL
– ARROYO
G.R. No. 191002, March 17, 2010
FACTS: The compulsory retirement of Chief Justice Reynato S. Puno by May 17, 2010 occurs just days after the
coming presidential elections on May 10, 2010.
These cases trace their genesis to the controversy that has arisen from the forthcoming compulsory retirement of
Chief Justice Puno on May 17, 2010, or seven days after the presidential election. Under Section 4(1), in relation
to Section 9, Article VIII, that “vacancy shall be filled within ninety days from the occurrence thereof” from a “list of
at least three nominees prepared by the Judicial and Bar Council for every vacancy.” Also considering that
Section 15, Article VII (Executive Department) of the Constitution prohibits the President or Acting President from
making appointments within two months immediately before the next presidential elections and up to the end of
his term, except temporary appointments to executive positions when continued vacancies therein will prejudice
public service or endanger public safety.
The JBC, in its en banc meeting of January 18, 2010, unanimously agreed to start the process of filling up the
position of Chief Justice.
Conformably with its existing practice, the JBC “automatically considered” for the position of Chief Justice the five
most senior of the Associate Justices of the Court, namely: Associate Justice Antonio T. Carpio; Associate Justice
Renato C. Corona; Associate Justice Conchita Carpio Morales; Associate Justice Presbitero J. Velasco, Jr.; and
Associate Justice Antonio Eduardo B. Nachura. However, the last two declined their nomination through letters
dated January 18, 2010 and January 25, 2010, respectively.

The OSG contends that the incumbent President may appoint the next Chief Justice, because the prohibition
under Section 15, Article VII of the Constitution does not apply to appointments in the Supreme Court. It argues
that any vacancy in the Supreme Court must be filled within 90 days from its occurrence, pursuant to Section 4(1),
Article VIII of the Constitution; that had the framers intended the prohibition to apply to Supreme Court
appointments, they could have easily expressly stated so in the Constitution, which explains why the prohibition
found in Article VII (Executive Department) was not written in Article VIII (Judicial Department); and that the
framers also incorporated in Article VIII ample restrictions or limitations on the President’s power to appoint
members of the Supreme Court to ensure its independence from “political vicissitudes” and its “insulation from
political pressures,” such as stringent qualifications for the positions, the establishment of the JBC, the specified
period within which the President shall appoint a Supreme Court Justice.
A part of the question to be reviewed by the Court is whether the JBC properly initiated the process, there being
an insistence from some of the oppositors-intervenors that the JBC could only do so once the vacancy has
occurred (that is, after May 17, 2010). Another part is, of course, whether the JBC may resume its process until
the short list is prepared, in view of the provision of Section 4(1), Article VIII, which unqualifiedly requires the
President to appoint one from the short list to fill the vacancy in the Supreme Court (be it the Chief Justice or an
Associate Justice) within 90 days from the occurrence of the vacancy.
ISSUE: Whether the incumbent President can appoint the successor of Chief Justice Puno upon his retirement.
HELD:
Prohibition under Section 15, Article VII does not apply to appointments to fill a vacancy in the Supreme Court or
to other appointments to the Judiciary.
Two constitutional provisions are seemingly in conflict.
The first, Section 15, Article VII (Executive Department), provides: Section 15. Two months immediately before
the next presidential elections and up to the end of his term, a President or Acting President shall not make
appointments, except temporary appointments to executive positions when continued vacancies therein will
prejudice public service or endanger public safety.
The other, Section 4 (1), Article VIII (Judicial Department), states: Section 4. (1). The Supreme Court shall be
composed of a Chief Justice and fourteen Associate Justices. It may sit en banc or in its discretion, in division of
three, five, or seven Members. Any vacancy shall be filled within ninety days from the occurrence thereof.
Had the framers intended to extend the prohibition contained in Section 15, Article VII to the appointment of
Members of the Supreme Court, they could have explicitly done so. They could not have ignored the meticulous
ordering of the provisions. They would have easily and surely written the prohibition made explicit in Section 15,
Article VII as being equally applicable to the appointment of Members of the Supreme Court in Article VIII itself,
most likely in Section 4 (1), Article VIII. That such specification was not done only reveals that the prohibition
against the President or Acting President making appointments within two months before the next presidential
elections and up to the end of the President’s or Acting President’s term does not refer to the Members of the
Supreme Court.
Had the framers intended to extend the prohibition contained in Section 15, Article VII to the appointment of
Members of the Supreme Court, they could have explicitly done so. They could not have ignored the meticulous
ordering of the provisions. They would have easily and surely written the prohibition made explicit in Section 15,
Article VII as being equally applicable to the appointment of Members of the Supreme Court in Article VIII itself,
most likely in Section 4 (1), Article VIII. That such specification was not done only reveals that the prohibition
against the President or Acting President making appointments within two months before the next presidential
elections and up to the end of the President’s or Acting President’s term does not refer to the Members of the
Supreme Court.
Section 14, Section 15, and Section 16 are obviously of the same character, in that they affect the power of the
President to appoint. The fact that Section 14 and Section 16 refer only to appointments within the Executive
Department renders conclusive that Section 15 also applies only to the Executive Department. This conclusion is
consistent with the rule that every part of the statute must be interpreted with reference to the context, i.e. that
every part must be considered together with the other parts, and kept subservient to the general intent of the
whole enactment. It is absurd to assume that the framers deliberately situated Section 15 between Section 14 and
Section 16, if they intended Section 15 to cover all kinds of presidential appointments. If that was their intention in
respect of appointments to the Judiciary, the framers, if only to be clear, would have easily and surely inserted a
similar prohibition in Article VIII, most likely within Section 4 (1) thereof.

DRILON VS LIM
GR No. 112497
The principal issue in this case is the constitutionality of Section 187 of the Local Government Code. The
Secretary of Justice (on appeal to him of four oil companies and a taxpayer) declared Ordinance No. 7794
(Manila Revenue Code) null and void for non-compliance with the procedure in the enactment of tax ordinances
and for containing certain provisions contrary to law and public policy.
RTC’s Ruling:
1. The RTC revoked the Secretary’s resolution and sustained the ordinance. It declared Sec 187 of the LGC as
unconstitutional because it vests on the Secretary the power of control over LGUs in violation of the policy of local
autonomy mandated in the Constitution.
Petitioner’s Argument:
1. The annulled Section 187 is constitutional and that the procedural requirements for the enactment of tax
ordinances as specified in the Local Government Code had indeed not been observed. (Petition originally
dismissed by the Court due to failure to submit certified true copy of the decision, but reinstated it anyway.)
2. Grounds of non-compliance of procedure
a. No written notices as required by Art 276 of Rules of Local Government Code
b. Not published
c. Not translated to tagalog
Supreme Court’s Argument:
1. Section 187 authorizes the petitioner to review only the constitutionality or legality of tax ordinance. What he
found only was that it was illegal. That act is not control but supervision.
2. Control lays down the rules in the doing of act and if not followed order the act undone or re-done.
Supervision sees to it that the rules are followed.
3. Two grounds of declaring Manila Revenue Code null and void (1) inclusion of certain ultra vires provisions (2)
non-compliance with prescribed procedure in its enactment but were followed.
The requirements are upon approval of local development plans and public investment programs of LGU not to
tax ordinances.

JOSE VILLENA VS SECRETARY OF THE INTERIOR


67 Phil. 451 – Political Law – Control Power – Supervision – Suspension of a Local Government Official – Power
to suspend comes with the power to remove
Jose Villena was the then mayor of Makati in the 1930s. After investigation, the Secretary of Interior
recommended the suspension of Villena with the Office of the president who approved the same. The Secretary
then suspended Villena. Villena averred claiming that the Secretary has no jurisdiction over the matter. The power
or jurisdiction is lodged in the local government [the governor] pursuant to sec 2188 of the Administrative Code.
Further, even if the respondent Secretary of the Interior has power of supervision over local governments, that
power, according to the constitution, must be exercised in accordance with the provisions of law and the
provisions of law governing trials of charges against elective municipal officials are those contained in sec 2188 of
the Administrative Code as amended. In other words, the Secretary of the Interior must exercise his supervision
over local governments, if he has that power under existing law, in accordance with sec 2188 of the
Administrative Code, as amended, as the latter provisions govern the procedure to be followed in suspending and
punishing elective local officials while sec 79 (C) of the Administrative Code is the genera law which must yield to
the special law.
ISSUE: Whether or not the Secretary of Interior can suspend an LGU official under investigation.
HELD: Yes.
There is no clear and express grant of power to the secretary to suspend a mayor of a municipality who is under
investigation. On the contrary, the power appears lodged in the provincial governor by sec 2188 of the
Administrative Code which provides that “The provincial governor shall receive and investigate complaints made
under oath against municipal officers for neglect of duty, oppression, corruption or other form of maladministration
of office, and conviction by final judgment of any crime involving moral turpitude“.

The fact, however, that the power of suspension is expressly granted by sec 2188 of the Administrative Code to
the provincial governor does not mean that the grant is necessarily exclusive and precludes the Secretary of the
Interior from exercising a similar power. For instance, Villena admitted in the oral argument that the President of
the Philippines may himself suspend the petitioner from office in virtue of his greater power of removal (sec. 2191,
as amended, Administrative Code) to be exercised conformably to law. Indeed, if the President could, in the
manner prescribed by law, remove a municipal official; it would be a legal incongruity if he were to be devoid of
the lesser power of suspension. And the incongruity would be more patent if, possessed of the power both to
suspend and to remove a provincial official (sec. 2078, Administrative Code), the President were to be without the
power to suspend a municipal official. The power to suspend a municipal official is not exclusive. Preventive
suspension may be issued to give way for an impartial investigation.

LACSON-MAGALLANES CO., INC. VS JOSE PAÑO


November 3, 2011
21 SCRA 895 – Political Law – Delegation of Control Power to the Executive Secretary
Jose Magallanes was permitted to use and occupy a land used for pasture in Davao. The said land was a forest
zone which was later declared as an agricultural zone. Magallanes then ceded his rights to Lacson-Magallanes
Co., Inc. (LMC) of which he is a co-owner.
Jose Paño was a farmer who asserted his claim over the same piece of land. The Director of Lands denied
Paño’s request. The Secretary of Agriculture likewise denied his petition hence it was elevated to the Office of
the President.
Executive Secretary Juan Pajo ruled in favor of Paño. LMC averred that the earlier decision of the Secretary of
Agriculture is already conclusive hence beyond appeal. He also averred that the decision of the Executive
Secretary is an undue delegation of power. The Constitution, LMC asserts, does not contain any provision
whereby the presidential power of control may be delegated to the Executive Secretary. It is argued that it is the
constitutional duty of the President to act personally upon the matter.
ISSUE: Whether or not the power of control may be delegated to the Executive Secretary.
HELD: Yes. It is true that as a rule, the President must exercise his constitutional powers in person. However, the
president may delegate certain powers to the Executive Secretary at his discretion. The president may delegate
powers which are not required by the Constitution for him to perform personally. The reason for this allowance is
the fact that the resident is not expected to perform in person all the multifarious executive and administrative
functions. The office of the Executive Secretary is an auxiliary unit which assists the President. The rule which
has thus gained recognition is that “under our constitutional setup the Executive Secretary who acts for and in
behalf and by authority of the President has an undisputed jurisdiction to affirm, modify, or even reverse any
order” that the Secretary of Agriculture and Natural Resources, including the Director of Lands, may issue.
The act of the Executive Secretary, acting as the alter ego of the President, shall remain valid until reversed,
disapproved, or reprobated by the President. In this case, no reprobation was made hence the decision granting
the land to Paño cannot be reversed.

CITY OF ILIGAN v DIRECTOR OF LANDS (Art. 7 Sec. 17)


1. President issued Proclamation 335:
a. Withdrawingcertain parcels of public land in Iligan from sale or settlement and
b. Reserving such for the use of NPC (Nat’l Power Corporation)
2. By virtue of said proclamation,NPC constructed a fertilizer plant named “Maria Cristina”
3. Later, NPC:
a. Sold the fertilizer plant to “Marcelo Tire and Rubber Corp” with all the machineries, right of occupancy,
and use of land
b. Covenanted to collaborate with DANR in facilitating sale and right to lease for at least 25 years, the
lands where plant is erected
4. Proclamation 20 and 198 were issued:
a. Proc. 20 – excluding from operation of Proc. 335 certain areas occupied by “Ma. Cristina” and
Employees Housing and declaring such lands for OPEN DISPOSITION
b. Proc. 198 – changing the technical description of said areas (6 lots)
5. “Marcelo Steel” and “Ma. Cristina” filed a Msc. Sales Application with the Bureau of Lands
a. “Marcelo Tire” and “Ma. Cristina” are sister corporations.
b. Purchaser was “Marcelo Tire” but another sister corp. “Marcelo Steel” operated said plant
6. In the notice of sale issued in Manila, Director of Lands advised that Bureau will sell in an auction said lands
of “Marcelo Steel”
7. President then issued Proc. 469 – excluding from the reservation made in favor to NPC certain lands in Iligan
(Lot 1, 1-a, 3, and 4) and DONATING said lands in favor of Iligan City.
8. Mayor of Iligan wrote to Director of Lands informing him that City is the owner of said lands and foreshores in
auction.
9. BUT no action was taken on said request for exclusion and so City filed a complaint for injunction in CFI
against Director. Injunction temporarily issued.
10. Pending case, President Marcos issued Proc. 94 – excluding from the donation in Proc. 469 certain lands
(Lot 1-a, 2-a, and 3) and declaring same for open disposition.
11. CFI dismissed the complaint of City and dissolved injunction. Hence, this appeal.

Issue: WON President has the authority to grant a portion of public domain to any government like the City of
Iligan.
Held: YES
1. Section 60 of Public Land Act states that tracts of land can be disposed of by grant, donationor transfer made
to a province, municipality, branch, or subdivision of government for purposes conducive to public interest.
a. Who has authority to donate? Secretary of Agriculture and National Resources through Director of
Lands (Sec 60)
2. Can President donate instead of Secretary and Director? YES
a. Director has direct executive control of lands (e.g. lease, sale, concession, disposition of land of
public domain)
b. Director SUBJECT to control of Secretary of Agriculture.
c. Secretary’s control is SUBJECT to control of PRESIDENT
d. Under Art VII Sec 17: President shall control ALL executive departments, bureaus, and offices.
e. Hence, President has the same authority to dispose of portions of public domain as his subordinates.
f. Such authority to dispose is also granted to the President under Section 69 of the Public Land Act.
3. Since, President has the authority to donate lands of public domain for residential, commercial, & industrial
purposes. Questioned Proclamation469 is VALIDand binding:
a. Ownership of lands now vested in City of Iligan.
b. Mayor of City upon proclamation immediately had the lots surveyed and entered into negotiation with
National Investment and Development Corp. and those interested in developing the Coco-Chemical
Plant in order to accelerate economic expansion in the City.
4. Proclamation 94 is NULL and VOID as said parcels had been segregated and had become property of Iligan.
5. Decision of CFI REVERSED.

GASCON V ARROYO
FACTS:
The Lopez family owned 2 television stations. When martial law was declared, the stations were seized. After the
Marcos
regime was toppled, the Presidential Committee on Good Governance (PCGG) sequestered said stations. Mr
Lopez requested the return of the stations. An agreement to arbitrate was entered into regarding this matter.
Thereupon, petitioners as taxpayers sought to set aside the agreement to arbitrate.
ISSUE:
Whether petitioners as taxpayers have legal standing to sue
RULING:
No. Petitioners have not shown that they have a legal interest in the TV stations and that they would be adversely
affected if and when the station is returned to the Lopez family. The present case is not an action to question the
constitutionality or validity of a law.

KILUSANG MAYO UNO LABOR CENTER VS. GARCIA, JR.,

An administrative body may implement broad policies laid down in a statute by “filling in” the details which the
Legislature may neither have time nor competence to provide. However, nowhere under the aforesaid provisions
of law are the regulatory bodies authorized to delegate that power to a common carrier, a transport operator or
other public service.

Exception To The Exception:


The authority given by the LTFRB to the provincial bus operators to set a fare range over and above the
authorized existing fare is illegal and invalid as it is tantamount to an undue delegation of legislative authority.
Potestas delegate non delegari potest. What has been delegated cannot be delegated. This doctrine is based on
the ethical principle that such a delegated power constitutes not only a right but a duty to be performed by the
delegate through the instrumentality of his own judgment and not through the intervening mind of another. A
further delegation of such power would indeed constitute a negation of the duty in violation of the trust reposed in
the delegate mandated to discharge it directly. The policy of allowing the provincial bus operators to change and
increase their fares at will would result not only to a chaotic situation but to an anarchic state of affairs. This would
leave the riding public at the mercy of transport operators who may increase fares every hour, every day, every
month or every year, whenever it pleases them or whenever they deem it “necessary” to do so.

IBP VS. ZAMORA


G.R. No.141284, August 15, 2000

Facts: Invoking his powers as Commander-in-Chief under Sec. 18, Art. VII of the Constitution, the President
directed the AFP Chief of Staff and PNP Chief to coordinate with each other for the proper deployment and
utilization of the Marines to assist the PNP in preventing or suppressing criminal or lawless violence. The
President declared that the services of the Marines in the anti-crime campaign are merely temporary in nature
and for a reasonable period only, until such time when the situation shall have improved. The IBP filed a petition
seeking to declare the deployment of the Philippine Marines null and void and unconstitutional.

Issues:
(1) Whether or not the President’s factual determination of the necessity of calling the armed forces is subject to
judicial review
(2) Whether or not the calling of the armed forces to assist the PNP in joint visibility patrols violates the
constitutional provisions on civilian supremacy over the military and the civilian character of the PNP

Held:
When the President calls the armed forces to prevent or suppress lawless violence, invasion or rebellion, he
necessarily exercises a discretionary power solely vested in his wisdom. Under Sec. 18, Art. VII of the
Constitution, Congress may revoke such proclamation of martial law or suspension of the privilege of the writ of
habeas corpus and the Court may review the sufficiency of the factual basis thereof. However, there is no such
equivalent provision dealing with the revocation or review of the President’s action to call out the armed forces.
The distinction places the calling out power in a different category from the power to declare martial law and
power to suspend the privilege of the writ of habeas corpus, otherwise, the framers of the Constitution would have
simply lumped together the 3 powers and provided for their revocation and review without any qualification.
The reason for the difference in the treatment of the said powers highlights the intent to grant the President the
widest leeway and broadest discretion in using the power to call out because it is considered as the lesser and
more benign power compared to the power to suspend the privilege of the writ of habeas corpus and the power to
impose martial law, both of which involve the curtailment and suppression of certain basic civil rights and
individual freedoms, and thus necessitating safeguards by Congress and review by the Court.

In view of the constitutional intent to give the President full discretionary power to determine the necessity of
calling out the armed forces, it is incumbent upon the petitioner to show that the President’s decision is totally
bereft of factual basis. The present petition fails to discharge such heavy burden, as there is no evidence to
support the assertion that there exists no justification for calling out the armed forces.

The Court disagrees to the contention that by the deployment of the Marines, the civilian task of law enforcement
is “militarized” in violation of Sec. 3, Art. II of the Constitution. The deployment of the Marines does not constitute
a breach of the civilian supremacy clause. The calling of the Marines constitutes permissible use of military assets
for civilian law enforcement. The local police forces are the ones in charge of the visibility patrols at all times, the
real authority belonging to the PNP

Moreover, the deployment of the Marines to assist the PNP does not unmake the civilian character of the police
force. The real authority in the operations is lodged with the head of a civilian institution, the PNP, and not with the
military. Since none of the Marines was incorporated or enlisted as members of the PNP, there can be no
appointment to civilian position to speak of. Hence, the deployment of the Marines in the joint visibility patrols
does not destroy the civilian character of the PNP.
OLAGUER VS MILITARY COMMISSION
Habeas Corpus
In 1979, Olaguer and some others were detained by military personnel and they were placed in Camp Bagong
Diwa. Logauer and his group are all civilians. They were charged with (1) unlawful possession of explosives and
incendiary devices; (2) conspiracy to assassinate President and Mrs. Marcos; (3) conspiracy to assassinate
cabinet members Juan Ponce Enrile, Francisco Tatad and Vicente Paterno; (4) conspiracy to assassinate
Messrs. Arturo Tangco, Jose Roño and Onofre Corpus; (5) arson of nine buildings; (6) attempted murder of
Messrs. Leonardo Perez, Teodoro Valencia and Generals Romeo Espino and Fabian Ver; and (7) conspiracy and
proposal to commit rebellion, and inciting to rebellion. On August 19, 1980, the petitioners went to the SC and
filed the instant Petition for prohibition and habeas corpus.
ISSUE: Whether or not the petition for habeas corpus be granted.
HELD: The petition for habeas corpus has become moot and academic because by the time the case reached
the SC Olaguer and his companions were already released from military confinement. “When the release of the
persons in whose behalf the application for a writ of habeas corpus was filed is effected, the Petition for the
issuance of the writ becomes moot and academic. 18 Inasmuch as the herein petitioners have been released
from their confinement in military detention centers, the instant Petitions for the issuance of a writ of habeas
corpus should be dismissed for having become moot and academic.” But the military court created to try the case
of Olaguer (and the decision it rendered) still continues to subsist.
ISSUE2: The issue is then shifted to: Whether or not a military tribunal has the jurisdiction to try civilians while the
civil courts are open and functioning.
HELD: The SC nullified for lack of jurisdiction all decisions rendered by the military courts or tribunals during the
period of martial law in all cases involving civilian defendants. A military commission or tribunal cannot try and
exercise jurisdiction, even during the period of martial law, over civilians for offenses allegedly committed by them
as long as the civil courts are open and functioning, and that any judgment rendered by such body relating to a
civilian is null and void for lack of jurisdiction on the part of the military tribunal concerned.

SANLAKAS VS. EXECUTIVE SECRETARY


421 SCRA 656 G.R. No. 159085
February 3, 2004
Facts: During the wee hours of July 27, 2003, some three-hundred junior officers and enlisted men of the AFP,
acting upon instigation, command and direction of known and unknown leaders have seized the Oakwood
Building in Makati. Publicly, they complained of the corruption in the AFP and declared their withdrawal of support
for the government, demanding the resignation of the President, Secretary of Defense and the PNP Chief. These
acts constitute a violation of Article 134 of the Revised Penal Code, and by virtue of Proclamation No. 427 and
General Order No. 4, the Philippines was declared under the State of Rebellion. Negotiations took place and the
officers went back to their barracks in the evening of the same day. On August 1, 2003, both the Proclamation
and General Orders were lifted, and Proclamation No. 435, declaring the Cessation of the State of Rebellion was
issued.
In the interim, however, the following petitions were filed: (1) SANLAKAS AND PARTIDO NG MANGGAGAWA
VS. EXECUTIVE SECRETARY, petitioners contending that Sec. 18 Article VII of the Constitution does not require
the declaration of a state of rebellion to call out the AFP, and that there is no factual basis for such proclamation.
(2)SJS Officers/Members v. Hon. Executive Secretary, et al, petitioners contending that the proclamation is a
circumvention of the report requirement under the same Section 18, Article VII, commanding the President to
submit a report to Congress within 48 hours from the proclamation of martial law. Finally, they contend that the
presidential issuances cannot be construed as an exercise of emergency powers as Congress has not delegated
any such power to the President. (3) Rep. Suplico et al. v. President Macapagal-Arroyo and Executive Secretary
Romulo, petitioners contending that there was usurpation of the power of Congress granted by Section 23 (2),
Article VI of the Constitution. (4) Pimentel v. Romulo, et al, petitioner fears that the declaration of a state of
rebellion "opens the door to the unconstitutional implementation of warrantless arrests" for the crime of rebellion.
Issue:
Whether or Not Proclamation No. 427 and General Order No. 4 are constitutional?
Whether or Not the petitioners have a legal standing or locus standi to bring suit?
Held: The Court rendered that the both the Proclamation No. 427 and General Order No. 4 are constitutional.
Section 18, Article VII does not expressly prohibit declaring state or rebellion. The President in addition to its
Commander-in-Chief Powers is conferred by the Constitution executive powers. It is not disputed that the
President has full discretionary power to call out the armed forces and to determine the necessity for the exercise
of such power. While the Court may examine whether the power was exercised within constitutional limits or in a
manner constituting grave abuse of discretion, none of the petitioners here have, by way of proof, supported their
assertion that the President acted without factual basis. The issue of the circumvention of the report is of no merit
as there was no indication that military tribunals have replaced civil courts or that military authorities have taken
over the functions of Civil Courts. The issue of usurpation of the legislative power of the Congress is of no
moment since the President, in declaring a state of rebellion and in calling out the armed forces, was merely
exercising a wedding of her Chief Executive and Commander-in-Chief powers. These are purely executive
powers, vested on the President by Sections 1 and 18, Article VII, as opposed to the delegated legislative powers
contemplated by Section 23 (2), Article VI. The fear on warrantless arrest is unreasonable, since any person may
be subject to this whether there is rebellion or not as this is a crime punishable under the Revised Penal Code,
and as long as a valid warrantless arrest is present.
Legal standing or locus standi has been defined as a personal and substantial interest in the case such that the
party has sustained or will sustain direct injury as a result of the governmental act that is being challenged. The
gist of the question of standing is whether a party alleges "such personal stake in the outcome of the controversy
as to assure that concrete adverseness which sharpens the presentation of Issue upon which the court depends
for illumination of difficult constitutional questions. Based on the foregoing, petitioners Sanlakas and PM, and SJS
Officers/Members have no legal standing to sue. Only petitioners Rep. Suplico et al. and Sen. Pimentel, as
Members of Congress, have standing to challenge the subject issuances. It sustained its decision in Philippine
Constitution Association v. Enriquez, that the extent the powers of Congress are impaired, so is the power of each
member thereof, since his office confers a right to participate in the exercise of the powers of that institution.

GUDANI VS. SENGA


GR No. 170165, August 15, 2006 [Article VI Sec. 22: Congress' Power of Inquiry; Legislative Investigation]
FACTS:
The Senate invited Gen. Gudani and Lt. Col. Balutan to clarify allegations of 2004 election fraud and the surfacing
of the “Hello Garci” tapes. PGMA issued EO 464 enjoining officials of the executive department including the
military establishment from appearing in any legislative inquiry without her consent. AFP Chief of Staff Gen.
Senga issued a Memorandum, prohibiting Gen. Gudani, Col. Balutan et al from appearing before the Senate
Committee without Presidential approval. However, the two appeared before the Senate in spite the fact that a
directive has been given to them. As a result, the two were relieved of their assignments for allegedly violating the
Articles of War and the time honoured principle of the “Chain of Command.” Gen. Senga ordered them to be
subjected before the General Court Martial proceedings for willfuly violating an order of a superior officer.
ISSUE:
Whether or not the President has the authority to issue an order to the members of the AFP preventing them from
testifying before a legislative inquiry.
RULING:
Yes. The SC hold that President has constitutional authority to do so, by virtue of her power as commander-in-
chief, and that as a consequence a military officer who defies such injunction is liable under military justice. At the
same time, any chamber of Congress which seeks the appearance before it of a military officer against the
consent of the President has adequate remedies under law to compel such attendance. Any military official whom
Congress summons to testify before it may be compelled to do so by the President. If the President is not so
inclined, the President may be commanded by judicial order to compel the attendance of the military officer. Final
judicial orders have the force of the law of the land which the President has the duty to faithfully execute.
SC ruled in Senate v. Ermita that the President may not issue a blanket requirement of prior consent on executive
officials summoned by the legislature to attend a congressional hearing. In doing so, the Court recognized the
considerable limitations on executive privilege, and affirmed that the privilege must be formally invoked on
specified grounds. However, the ability of the President to prevent military officers from testifying before Congress
does not turn on executive privilege, but on the Chief Executive’s power as commander-in-chief to control the
actions and speech of members of the armed forces. The President’s prerogatives as commander-in-chief are not
hampered by the same limitations as in executive privilege.
At the same time, the refusal of the President to allow members of the military to appear before Congress is still
subject to judicial relief. The Constitution itself recognizes as one of the legislature’s functions is the conduct of
inquiries in aid of legislation. Inasmuch as it is ill-advised for Congress to interfere with the President’s power as
commander-in-chief, it is similarly detrimental for the President to unduly interfere with Congress’s right to conduct
legislative inquiries. The impasse did not come to pass in this petition, since petitioners testified anyway despite
the presidential prohibition. Yet the Court is aware that with its pronouncement today that the President has the
right to require prior consent from members of the armed forces, the clash may soon loom or actualize.
The duty falls on the shoulders of the President, as commander-in-chief, to authorize the appearance of the
military officers before Congress. Even if the President has earlier disagreed with the notion of officers appearing
before the legislature to testify, the Chief Executive is nonetheless obliged to comply with the final orders of the
courts.

RANDOLF DAVID VS PRESIDENT GLORIA MACAPAGAL-ARROYO


489 SCRA 160 – Political Law – The Executive Branch – Presidential Proclamation 1017 – Take Care Clause –
Take Over Power – Calling Out Power
Bill of Rights – Freedom of Speech – Overbreadth
In February 2006, due to the escape of some Magdalo members and the discovery of a plan (Oplan Hackle I) to
assassinate the president, then president Gloria Macapagal-Arroyo (GMA) issued Presidential Proclamation 1017
(PP1017) and is to be implemented by General Order No. 5 (GO 5). The said law was aimed to suppress
lawlessness and the connivance of extremists to bring down the government.
Pursuant to such PP, GMA cancelled all plans to celebrate EDSA I and at the same time revoked all permits
issued for rallies and other public organization/meeting. Notwithstanding the cancellation of their rally permit,
Kilusang Mayo Uno (KMU) head Randolf David proceeded to rally which led to his arrest.
Later that day, the Daily Tribune, which Cacho-Olivares is the editor, was raided by the CIDG and they seized and
confiscated anti-GMA articles and write ups. Later still, another known anti-GMA news agency (Malaya) was
raided and seized. On the same day, Beltran of Anakpawis, was also arrested. His arrest was however grounded
on a warrant of arrest issued way back in 1985 for his actions against Marcos. His supporters cannot visit him in
jail because of the current imposition of PP 1017 and GO 5.
In March, GMA issued PP 1021 which declared that the state of national emergency ceased to exist. David and
some opposition Congressmen averred that PP1017 is unconstitutional for it has no factual basis and it cannot be
validly declared by the president for such power is reposed in Congress. Also such declaration is actually a
declaration of martial law. Olivares-Cacho also averred that the emergency contemplated in the Constitution are
those of natural calamities and that such is an overbreadth. Petitioners claim that PP 1017 is an overbreadth
because it encroaches upon protected and unprotected rights. The Sol-Gen argued that the issue has become
moot and academic by reason of the lifting of PP 1017 by virtue of the declaration of PP 1021. The Sol-Gen
averred that PP 1017 is within the president’s calling out power, take care power and take over power.
ISSUE: Whether or not PP 1017 and GO 5 is constitutional.
HELD: PP 1017 and its implementing GO are partly constitutional and partly unconstitutional.
The issue cannot be considered as moot and academic by reason of the lifting of the questioned PP. It is still in
fact operative because there are parties still affected due to the alleged violation of the said PP. Hence, the SC
can take cognition of the case at bar. The SC ruled that PP 1017 is constitutional in part and at the same time
some provisions of which are unconstitutional. The SC ruled in the following way;
Resolution by the SC on the Factual Basis of its declaration
The petitioners were not able to prove that GMA has no factual basis in issuing PP 1017 and GO 5. A reading of
the Solicitor General’s Consolidated Comment and Memorandum shows a detailed narration of the events leading
to the issuance of PP 1017, with supporting reports forming part of the records. Mentioned are the escape of the
Magdalo Group, their audacious threat of the Magdalo D-Day, the defections in the military, particularly in the
Philippine Marines, and the reproving statements from the communist leaders. There was also the Minutes of the
Intelligence Report and Security Group of the Philippine Army showing the growing alliance between the NPA and
the military. Petitioners presented nothing to refute such events. Thus, absent any contrary allegations, the
Court is convinced that the President was justified in issuing PP 1017 calling for military aid. Indeed, judging the
seriousness of the incidents, GMA was not expected to simply fold her arms and do nothing to prevent or
suppress what she believed was lawless violence, invasion or rebellion. However, the exercise of such power or
duty must not stifle liberty.
Resolution by the SC on the Overbreadth Theory
First and foremost, the overbreadth doctrine is an analytical tool developed for testing ‘on their faces’ statutes in
free speech cases. The 7 consolidated cases at bar are not primarily ‘freedom of speech’ cases. Also, a plain
reading of PP 1017 shows that it is not primarily directed to speech or even speech-related conduct. It is actually
a call upon the AFP to prevent or suppress all forms of lawless violence. Moreover, the overbreadth doctrine is
not intended for testing the validity of a law that ‘reflects legitimate state interest in maintaining comprehensive
control over harmful, constitutionally unprotected conduct.’ Undoubtedly, lawless violence, insurrection and
rebellion are considered ‘harmful’ and ‘constitutionally unprotected conduct.’ Thus, claims of facial overbreadth
are entertained in cases involving statutes which, by their terms, seek to regulate only ‘spoken words’ and again,
that ‘overbreadth claims, if entertained at all, have been curtailed when invoked against ordinary criminal laws that
are sought to be applied to protected conduct.’ Here, the incontrovertible fact remains that PP 1017 pertains to a
spectrum of conduct, not free speech, which is manifestly subject to state regulation.
Resolution by the SC on the Calling Out Power Doctrine
On the basis of Sec 17, Art 7 of the Constitution, GMA declared PP 1017. The SC considered the President’s
‘calling-out’ power as a discretionary power solely vested in his wisdom, it stressed that ‘this does not prevent an
examination of whether such power was exercised within permissible constitutional limits or whether it was
exercised in a manner constituting grave abuse of discretion. The SC ruled that GMA has validly declared PP
1017 for the Constitution grants the President, as Commander-in-Chief, a ‘sequence’ of graduated powers. From
the most to the least benign, these are: the calling-out power, the power to suspend the privilege of the writ of
habeas corpus, and the power to declare Martial Law. The only criterion for the exercise of the calling-out power
is that ‘whenever it becomes necessary,’ the President may call the armed forces ‘to prevent or suppress lawless
violence, invasion or rebellion.’ And such criterion has been met.
Resolution by the SC on the Take Care Doctrine
Pursuant to the 2nd sentence of Sec 17, Art 7 of the Constitution (He shall ensure that the laws be faithfully
executed.) the president declared PP 1017. David et al averred that PP 1017 however violated Sec 1, Art 6 of the
Constitution for it arrogated legislative power to the President. Such power is vested in Congress. They assail the
clause ‘to enforce obedience to all the laws and to all decrees, orders and regulations promulgated by me
personally or upon my direction.’ The SC noted that such provision is similar to the power that granted former
President Marcos legislative powers (as provided in PP 1081). The SC ruled that the assailed PP 1017 is
unconstitutional insofar as it grants GMA the authority to promulgate ‘decrees.’ Legislative power is peculiarly
within the province of the Legislature. Sec 1, Article 6 categorically states that ‘[t]he legislative power shall be
vested in the Congress of the Philippines which shall consist of a Senate and a House of Representatives.’ To be
sure, neither Martial Law nor a state of rebellion nor a state of emergency can justify GMA’[s exercise of
legislative power by issuing decrees. The president can only “take care” of the carrying out of laws but cannot
create or enact laws.
Resolution by the SC on the Take Over Power Doctrine
The president cannot validly order the taking over of private corporations or institutions such as the Daily Tribune
without any authority from Congress. On the other hand, the word emergency contemplated in the constitution is
not limited to natural calamities but rather it also includes rebellion. The SC made a distinction; the president can
declare the state of national emergency but her exercise of emergency powers does not come automatically after
it for such exercise needs authority from Congress. The authority from Congress must be based on the following:
(1) There must be a war or other emergency.
(2) The delegation must be for a limited period only.
(3) The delegation must be subject to such restrictions as the Congress may prescribe.
(4) The emergency powers must be exercised to carry out a national policy declared by Congress.
Resolution by the SC on the Issue that PP 1017 is a Martial Law Declaration
The SC ruled that PP 1017 is not a Martial Law declaration and is not tantamount to it. It is a valid exercise of the
calling out power of the president by the president.

TORRES v. GONZALES
July 23, 1987 (G.R. No. 76872)
FACTS:
1978, Torres was convicted of estafa. In 1979, he was pardoned by the president w/ the condition that he shall
not violate any penal laws again. Should this condition be violated, he will be proceeded against in the manner
prescribed by law. Petitioner accepted the conditional pardon and was consequently released from confinement.
In 1982, Torres was charged with multiple crimes of estafa. In 1986, Gonzales petitioned for the cancellation of
Torres’ pardon. Hence, the president cancelled the pardon. Torres appealed the issue before the SC averring that
the Exec Dep’t erred in convicting him for violating the conditions of his pardon because the estafa charges
against him were not yet final and executory as they were still on appeal.
ISSUE: whether or not conviction of a crime by final judgment of a court is necessary before the petitioner can be
validly rearrested and recommitted for violation of the terms of his conditional pardon and accordingly to serve the
balance of his original sentence.
HELD: In proceeding against a convict who has been conditionally pardoned and who is alleged to have
breached the conditions of his pardon, the Executive Department has two options: (1) Section 64 (i) of the
Revised Administrative Code, a purely executive act, not subject to judicial scrutiny, or (2) Article 159 of the
Revised Penal Code, a judicial act consisting of trial for and conviction of violation of a conditional pardon.
Where the President opts to proceed under Section 64 (i) of the Revised Administrative Code, no judicial
pronouncement of guilt of a subsequent crime is necessary, much less conviction therefor by final judgment of a
court, in order that a convict may be recommended for the violation of his conditional pardon.
Under art. 159 of the RPC, parolee or convict who is regarded as having violated the provisions thereof must be
charged, prosecuted and convicted by final judgment before he can be made to suffer the penalty prescribed.
In the case at bar, President has chosen to proceed against the petitioner under Section 64 (i) of the Revised
Administrative Code. That choice is an exercise of the President’s executive prerogative and is not subject to
judicial scrutiny.
*Who determines if violated? The PRESIDENT. When the person was conditionally pardoned it was a generous
exercise by the Chief Executive of his constitutional prerogative. The acceptance thereof by the convict or
prisoner carrie[d] with it the authority or power of the Executive to determine whether a condition or conditions of
the pardon has or have been violated. To no other department of the Government [has] such power been
intrusted.

MONSANTO VS. FACTORAN


Facts:
The Sandiganbayan convicted petitioner Salvacion A. Monsanto (then assistant treasurer of Calbayog City) of the
crime of estafa through falsification of public documents. She was sentenced to jail and to indemnify the
government in the sum of P4,892.50.The SC affirmed the decision. She then filed a motion for reconsideration but
while said motion was pending, she was extended by then President Marcos absolute pardon which she accepted
(at that time, the rule was that clemency could be given even before conviction). By reason of said pardon,
petitioner wrote the Calbayog City treasurer requesting that she be restored to her former post as assistant city
treasurer since the same was still vacant. Her letter was referred to the Minister of Finance who ruled that she
may be reinstated to her position without the necessity of a new appointment not earlier than the date she was
extended the absolute pardon.
Petitioner wrote the Ministry stressing that the full pardon bestowed on her has wiped out the crime which implies
that her service in the government has never been interrupted and therefore the date of her reinstatement should
correspond to the date of her preventive suspension; that she is entitled to backpay for the entire period of her
suspension; and that she should not be required to pay the proportionate share of the amount of P4,892.50
The Ministry referred the issue to the Office of the President. Deputy Executive Secretary Factoran denied
Monsanto’s request averring that Monsanto must first seek appointment and that the pardon does not reinstate
her former position.
Issues:
1. Is Monsanto entitled to backpay?
2. Is a public officer, who has been granted an absolute pardon by the Chief Executive, entitled to reinstatement
to her former position without need of a new appointment?
3. May petitioner be exempt from the payment of the civil indemnity imposed upon her by the sentence?
Held:
1. Pardon is defined as "an act of grace, proceeding from the power entrusted with the execution of the laws,
which exempts the individual, on whom it is bestowed, from the punishment the law inflicts for a crime he has
committed. It is the private, though official act of the executive magistrate, delivered to the individual for whose
benefit it is intended, and not communicated officially to the Court.
While a pardon has generally been regarded as blotting out the existence of guilt so that in the eye of the law the
offender is as innocent as though he never committed the offense, it does not operate for all purposes. The very
essence of a pardon is forgiveness or remission of guilt. Pardon implies guilt. It does not erase the fact of the
commission of the crime and the conviction thereof. It does not wash out the moral stain. It involves forgiveness
and not forgetfulness.
A pardon looks to the future. It is not retrospective. It makes no amends for the past. It affords no relief for what
has been suffered by the offender. It does not impose upon the government any obligation to make reparation for
what has been suffered. “Since the offense has been established by judicial proceedings, that which has been
done or suffered while they were in force is presumed to have been rightfully done and justly suffered, and no
satisfaction for it can be required.” This would explain why petitioner, though pardoned, cannot be entitled to
receive backpay for lost earnings and benefits.
2. The pardon granted to petitioner has resulted in removing her disqualification from holding public employment
but it cannot go beyond that. To regain her former post as assistant city treasurer, she must re-apply and undergo
the usual procedure required for a new appointment.
3. Civil liability arising from crime is governed by the Revised Penal Code. It subsists notwithstanding service of
sentence, or for any reason the sentence is not served by pardon, amnesty or commutation of sentence.
Petitioner's civil liability may only be extinguished by the same causes recognized in the Civil Code, namely:
payment, loss of the thing due, remission of the debt, merger of the rights of creditor and debtor, compensation
and novation. (Monsanto vs. Factoran, G.R. No. 78239, February 9, 1989)

PEOPLE VS. SALLE

Where the judgment of conviction is still pending appeal and has not yet therefore attained finality, as in the
instant case, executive clemency may not yet be granted to the appellant.

The acceptance of the pardon shall not operate as an abandonment or waiver of the appeal.

Facts:

On November 1991, Francisco Salle, Jr. and Ricky Mengote were convicted of the compound crime of murder
and destructive arson before the RTC of Quezon City. Salle and Mengote filed their Notice of Appeal which was
accepted by the Supreme Court on March 24, 1993.
In 1994, Salle filed an Urgent Motion to Withdraw Appeal. The Court required Salle's counsel, Atty. Ida May La'o
of the Free Legal Assistance Group (FLAG) to verify the voluntariness of the motion.

Atty. La'o manifested that Salle signed the motion without the assistance of counsel on his misimpression that the
motion was necessary for his early release from the New Bilibid Prison following the grant of a conditional pardon
by the President on December 9, 1993. She also stated that Mengote was also grantedconditional pardon and
that he immediately left for his province without consulting her. She prayed that the Court grant Salle's motion to
withdraw his appeal.

On March 23, 1994, the Court granted Salle's motion.

After taking into consideration Section 19, Article VII of the Constitution which provides that the President may,
except in cases of impeachment or as otherwise provided in the Constitution, grant pardon after conviction by
final judgment, the Court required (1) the Solicitor General and the counsel for accused-appellants to submit their
memoranda on the issue of the enforceability of the conditionalpardon and (2) the Presidential Committee for the
Grant of Bail, Release or Pardon to inform the Court why it recommended to the President the grant of
theconditional pardon despite the pendency of the appeal.

In its Memorandum, the Office of the Solicitor General maintains that theconditional pardon granted
to appellant Mengote is unenforceable because the judgment of conviction is not yet final in view of the pendency
in this Court of his appeal.

On the other hand, the FLAG, through Atty. La'o, submits that the conditionalpardon extended to Mengote is valid
and enforceable. Citing Monsanto vs. Factoran, Jr., it argues that although Mengote did not file a motion to
withdraw the appeal, he was deemed to have abandoned the appeal by his acceptance of theconditional pardon
which resulted in the finality of his conviction.

Issue:

Whether or not a pardon granted to an accused during the pendency of his appeal from a judgment of conviction
by the trial court is enforceable.

Held:

Section 19, Article VII thereof reads as follows:

“Except in cases of impeachment, or as otherwise provided in this Constitution, the President may grant
reprieves, commutations, and pardons, and remit fines and forfeitures, after conviction by final judgment.

He shall also have the power to grant amnesty with the concurrence of a majority of all the Members of the
Congress.”

Where the pardoning power is subject to the limitation of conviction, it may be exercised at any time after
conviction even if the judgment is on appeal. It is, of course, entirely different where the requirement is " final
conviction, " as was mandated in the original provision of Section 14, Article IX of the 1973 Constitution, or
"conviction by final judgment," as presently prescribed in Section 19, Article VII of the 1987 Constitution. In such a
case, no pardon may be extended before a judgment of conviction becomes final.

A judgment of conviction becomes final (a) when no appeal is seasonably perfected, (b) when the accused
commences to serve the sentence, (c) when the right to appeal is expressly waived in writing, except where the
death penalty was imposed by the trial court, and (d) when the accused applies for probation, thereby waiving his
right to appeal. Where the judgment of conviction is still pending appeal and has not yet therefore attained finality,
as in the instant case, executive clemency may not yet be granted to the appellant.

The "conviction by final judgment" limitation under Section 19, Article VII of the present Constitution prohibits the
grant of pardon, whether full or conditional, to an accused during the pendency of his appeal from his conviction
by the trial court. Any application therefor, if one is made, should not be acted upon or the process toward its
grant should not be begun unless the appeal is withdrawn.Accordingly, the agencies or instrumentalities of the
Government concerned must require proof from the accused that he has not appealed from his conviction or that
he has withdrawn his appeal. Such proof may be in the form of a certification issued by the trial court or
the appellate court, as the case may be.

The acceptance of the pardon shall not operate as an abandonment or waiver of the appeal, and the release of
an accused by virtue of a pardon, commutation of sentence, or parole before the withdrawal of an appeal shall
render those responsible therefor administratively liable. Accordingly, those in custody of the accused must not
solely rely on the pardon as a basis for the release of the accused from confinement.

WHEREFORE, counsel for accused-appellant Ricky Mengote y Cuntado is hereby given thirty (30) days from
notice hereof within which to secure from the latter the withdrawal of his appeal and to submit it to this Court.
The conditional pardon granted the said appellant shall be deemed to take effect only upon the grant of such
withdrawal. In case of non-compliance with this Resolution, the Director of the Bureau of Corrections must exert
every possible effort to take back into his custody the said appellant, for which purpose he may seek the
assistance of the Philippine National Police or the National Bureau of Investigation. (People vs. Francisco Salle,
Jr. and Ricky Mengote, G.R. No. 103567, December 4, 1995)

GARCIA VS COA
G.R. No. 75025 September 14, 1993
Facts:
Herein petitioner Vicente Garcia was employed as a Supervising lineman at the Bureau of Telecommunications.
He was accused of stealing some materials in their company. Thus, public respondents filed a criminal case
against him for qualified theft before a court and on the same ground respondents also filed an administrative
case in which petitioner was found guilty and was later dismissed from the service. With respect to the criminal
offense, petitioner was acquitted by the court due to insufficiency of evidence. Petitioner was then reinstated from
his work and is now claiming before the COA for his back salaries from the time of his dismissal up to present. But
COA on the other hand reluctantly denied his pleadings. Meanwhile, petitioner was extended an executive
clemency (absolute pardon) by the President. Still, respondent COA strongly refused to give due course to
petitioners claim.
Issue:
Whether or not respondent is entitled to the payment of back wages after having been reinstated pursuant to the
grant of executive clemency.
Holding:
The Court ruled initially by explaining the mandate of Sec 19 Article VII of the Constitution and further articulates
that the bestowal of executive clemency on petitioner in effect completely obliterated the adverse effects of the
administrative decision which found him guilty of dishonesty and ordered his separation from the service. This can
be inferred from the executive clemency itself exculpating petitioner from the administrative charge and thereby
directing his reinstatement, which is rendered automatic by the grant of the pardon. This signifies that petitioner
need no longer apply to be reinstated to his former employment; he is restored to his office ipso facto upon the
issuance of the clemency.
RODOLFO LLAMAS VS EXEC SEC ORBOS & MARIANO OCAMPO III
Pardon – Applicable to Administrative Cases
Ocampo III was the governor of Tarlac Province. Llamas together with some other complainants filed an
administrative case against Ocampo III for alleged acts constituting graft and corruption. Ocampo III was found
guilty. He was suspended for office for 90 days hence his vice governor, Llamas, assumed office. In not less than
30 days however, Ocampo III returned with an AO showing that he was pardoned hence he can resume office
without completing the 90 day suspension imposed upon him.
ISSUE: Whether or not pardon is applicable to administrative cases.
HELD: The SC held that pardon is applicable to Administrative cases. The SC does not clearly see any valid and
convincing reason why the President cannot grant executive clemency in administrative cases. It is a considered
view that if the President can grant reprieves, commutations and pardons, and remit fines and forfeitures in
criminal cases, with much more reason can she grant executive clemency in administrative cases, which are
clearly less serious than criminal offenses.

KAPUNAN VS COURT OF APPEALS

Facts: The petitioners, Edgardo Kapunan and Oscar Legaspi charges with the killing of KMU Chairman Rolando
Olalia and his driver Leonor Alay-ay.

On June 1986 Olalia and Alay-ay dead body was found. The murder case of Olalia is a controversial case during
that time, Oliala is a profile individual being the Chairman of the KMU at the time of his death..

On Nov. 1998, private respondents Feliciano Olalia and Perlina Alay-ay, filed a complaint letter to Department of
Justice for the alleged complex kidnapping and killing of Olalia and Alay-ay against Edgardo Kapunan and Oscar
Legaspi and other men and officers of Phil. National Police and the AFP.

Sec. Serafin Cuevas, the Secretary of Department of Justice, created a Panel that were tasked to conduct a
preliminary investigation of Olalia case.

The petitioner filed a motion to dismiss in Department of Justice on the ground that the Amnesty granted to them
by the National Amnesty Commission extinguishes their criminal liability under Proclamation 347 issued by Pres.
Fidel V. Ramos entitled, “Granting of Amnesty to the rebels, insurgents and all other persons, who may or may
be committed crimes against public order and crimes committed in furtherance of political ends.

The petitioners filed a motion for certiorari.

Issues: Whether or not the amnesty granted to Kapunan and Legaspi, extinguishes their criminal liability in Olalia
case?

Held/Ruling:

The Panel created by the Department of Justice refused to consider the defense of Amnesty of the petitioners on
the ground that the document presented pertaining to the Amnesty failed to show that the Olalia murder case was
one of the crimes for which the amnesty was applied for.

The Court of Appeals also dismissed the petition, finding no grave of abuse of discretion on the Panel created by
the DOJ, The Appelate Court refused to rule on the applicability of Amnesty issued to Kapunan and Legapi.

Finally the Supreme court s dismissed the petition for certiorari on the ground that the Amnesty granted to
Kapunan and Legaspi pertains only to the crimes against rebellion and not covered crime of murder Olalia and
Alay-ay case.
COMMISSIONER OF CUSTOMS & COLLECTOR OF CUSTOMS VS EASTERN SEA TRADING
3 SCRA 351 – Political Law – Constitutional Law – Treaties vs Executive Agreements
Eastern Sea Trading (EST) was a shipping company which imports from Japan onion and garlic into the
Philippines. In 1956, the Commissioner of Customs ordered the seizure and forfeiture of the import goods
because EST was not able to comply with Central Bank Circulars 44 and 45. The said circulars were pursuant to
Executive Order 328. On the other hand, EO 328 was the implementing law of the Trades and Financial
Agreements, an executive agreement, entered into between the Philippines and Japan. The said executive
agreement states, among others, that all import transactions between Japan and the Philippines should be
invoiced in dollar. In this case, the said items imported by EST from Japan were not invoiced in dollar.
EST questioned the validity of the said EO averring that the executive agreement that the EO was implementing
was never concurred upon by the Senate. The issue was elevated to the Court of Tax Appeals and the latter ruled
in favor of EST. The Commissioner appealed.
ISSUE: Whether or not the Executive Agreement is subject to the concurrence by the Senate.
HELD: No, Executive Agreements are not like treaties which are subject to the concurrence of at least 2/3 of the
members of the Senate. Agreements concluded by the President which fall short of treaties are commonly
referred to as executive agreements and are no less common in our scheme of government than are the more
formal instruments — treaties and conventions. They sometimes take the form of exchanges of notes and at other
times that of more formal documents denominated ‘agreements’ or ‘protocols’.
The point where ordinary correspondence between this and other governments ends and agreements — whether
denominated executive agreements or exchanges of notes or otherwise — begin, may sometimes be difficult of
ready ascertainment. It would be useless to undertake to discuss here the large variety of executive agreements
as such, concluded from time to time. Hundreds of executive agreements, other than those entered into under the
trade- agreements act, have been negotiated with foreign governments. . . . It would seem to be sufficient, in
order to show that the trade agreements under the act of 1934 are not anomalous in character, that they are not
treaties, and that they have abundant precedent in our history, to refer to certain classes of agreements
heretofore entered into by the Executive without the approval of the Senate.
They cover such subjects as the inspection of vessels, navigation dues, income tax on shipping profits, the
admission of civil aircraft, customs matters, and commercial relations generally, international claims, postal
matters, the registration of trade-marks and copyrights, etc. Some of them were concluded not by specific
congressional authorization but in conformity with policies declared in acts of Congress with respect to the
general subject matter, such as tariff acts; while still others, particularly those with respect to the settlement of
claims against foreign governments, were concluded independently of any legislation.

Anda mungkin juga menyukai